reading comprehension-the basics - byju's · even though reading comprehension topics are...

56
CSAT Reading Comprehension -Basics 1 © Copyright 2011.Think & Learn Pvt. Ltd. Copying or Reproduction without the prior permission of the author is prohibited READING COMPREHENSION-THE BASICS Contents 1.1 Introduction to Reading Comprehension 1.2 Components of Passages 1.3 Summary Techniques to improve Reading Comprehension 1.4 Practice Tests on Reading Comprehension

Upload: ngobao

Post on 19-Aug-2018

254 views

Category:

Documents


1 download

TRANSCRIPT

CSAT Reading Comprehension -Basics

1 © Copyright 2011.Think & Learn Pvt. Ltd. Copying or Reproduction without the prior permission of the author is prohibited

READING COMPREHENSION-THE BASICS

Contents

1.1 Introduction to Reading Comprehension

1.2 Components of Passages

1.3 Summary Techniques to improve Reading Comprehension

1.4 Practice Tests on Reading Comprehension

CSAT Reading Comprehension -Basics

2 © Copyright 2011.Think & Learn Pvt. Ltd. Copying or Reproduction without the prior permission of the author is prohibited

1.1 Introduction to Reading Comprehension

“Speed reading, high concentration and material retention are all linked. You can’t do one without the other very effectively. So, if you start reading fast, you are forced to concentrate and this increases retention. The high concentration that is induced when you try to read faster results in some interesting physiological changes in the brain where the memory of what you are reading gets etched more permanently than the times when you don’t have high concentration.”

What exactly IS Reading Comprehension?

Reading Comprehension is the ability to actively read the information (mostly passages) provided and assimilate the information to answer questions. One must be able to: Understand the crux of the passage Answer questions based on the text provided Develop a perspective on the tone & tenor of the narration Infer or deduce logical arguments

There is no denying that a good number of questions are from Reading Comprehension. Thus, you NEED to be a good reader. So how do you improve on your reading? Let’s look at some points that will help make you a better reader.

1.1.1 Be an active reader

This is probably the “oldest” and “best” advice you can get. Read as much as you can. Developing a habit of reading is of critical importance as it provides you with the basic foundation stone required for comprehension of passages in an effective manner.

1.1.2 Understanding vs. Speed

Separate the wheat from the chaff. This is basically the ability to distinguish sentences which are important, from sentences, which are too general. If a passage is talking about angiography and then there is an example provided on it then you don’t need to spend a lot of time with that sentence to understand the example. Hence while reading, keep an eye open for redundant sentences. Your reading motto should only be to grasp the idea, which is the theme of the passage. This can be done by skipping the unnecessary details and illustrations which are only there to reinforce the theme of the passage. The details, generally, tend to slow you down so it must be avoided.

1.1.3 Engage with the Passage

You simply cannot understand something which you loathe. So getting over the dread of the passage is a pre-requisite

to doing well in the questions. Actively involve yourself in the passage and try to simplify sentences.

1.1.4 Pace yourself well

As you read the passage, go faster after the first paragraph. Hold the framework of the passage in your memory as you

work your way through the passage. Do not get lost in details later on in the passage and do not try to master every bit

of the content.

1.1.5 Read Quality Articles

Always read articles that challenge your comprehension. It is not of much use if you keep on reading articles and passages that are straightforward and would only pamper your reading and comprehension skills. So, you must invest time in reading such articles that require greater skills of comprehension on your part.

CSAT Reading Comprehension -Basics

3 © Copyright 2011.Think & Learn Pvt. Ltd. Copying or Reproduction without the prior permission of the author is prohibited

1.1.6 Don’t just read! Write!

When the passage is tough, you should not just read it. You can build your comprehension more quickly and effectively

by writing down key elements of the passage. This will force you to read actively as opposed to passively. The “writing

down” should be as simple as possible & should allow you to grasp the simple story of the passage. Wherever possible,

use symbols and notations to save time;. Abbreviate as much as possible and focus on the real goal: creating true

comprehension quickly

1.1.7 Attempting Questions

There are a lot of schools of thought on how you should read a passage and answer questions. Some think you should read the questions first and then look for the answers; others feel you should read the passage thoroughly and then answer all the questions without looking back. We all know that there are two types of questions, direct and indirect. If you have read through the passage completely with in-depth understanding, then you may be able to answer all the questions correctly. If you read through the passage once with cursory understanding then you should be able to answer all main idea questions. You will mostly need to refer to the passage again for detail / direct questions. You would realize that a complete understanding of the entire passage is usually not required to answer most of the questions. You will learn to answer questions by just skimming through passages and learning to identify important sentences in class.

1.2 Components of Reading Comprehension

Even though Reading Comprehension topics are varied in topic and structure, the underlying components of a Reading Comprehension passage can be possibly classified into 1) The Main Point 2) Background 3) Support 4) Implications

1.2.1) The Main Point

As the name suggests, this is the point the author is trying to convey in the passage. Every passage contains a main point. Surprisingly, the main point will often be made explicit in a single sentence. The Main Point, is related to the passage’s purpose. The purpose of the passage is generally to convey the main point. The Background, Support and Implications are components which relate to the main point in some way.

1.2.2) Background

This is the information you need to understand after grasping the main point. The context and the basic facts about the passage are given in the background.

1.2.3) Support

The Support, is the evidence, assertions and opinions for the main point. The Background and the Support may be expressed together. A sentence can give the background information and support the main point at the same time.

1.2.4) Implications

These are the results from the Main Point. The author elaborates on the consequences of the Main Point. Implications may not always be present, but when they are, you can expect questions on them. 1.3 Summary Techniques to improve Reading Comprehension Develop a broad background. Broaden your background knowledge by reading newspapers, magazines and books. Become interested in world events. Know the structure of paragraphs. Paragraphs are constructed with a beginning (introduction), body and conclusion. Often, the first sentence will give an

CSAT Reading Comprehension -Basics

4 © Copyright 2011.Think & Learn Pvt. Ltd. Copying or Reproduction without the prior permission of the author is prohibited

overview that helps provide a framework for adding details. Also, look for transitional words, phrases or paragraphs that change the topic. Identify the type of reasoning. Does the author use cause and effect reasoning, hypothesis, model building, induction or deduction, systems thinking? Anticipate and predict. Try to anticipate the author and predict future ideas and questions. If you're right, this reinforces your understanding. If you're wrong, you make adjustments quickly. Always search for the main idea, instead of focusing on minor details While answering questions

Identify Key Words While answering specific questions, identify the KEY WORDS in the questions. Then, go back to the passage and find those key words.

Justify every word in the answer choice In the correct answer choice, every word must be completely true and within the scope of the passage. If you cannot justify every word in the answer choice, eliminate it!

Avoid extreme words if possible

Look out for words such as “all” and “never”. They tend to broaden the scope of an answer choice too much or make it too extreme. When you choose an “extreme answer choice”, the passage should back it completely. Look for the method of organization. Is the material organized chronologically, serially, logically, functionally, spatially or hierarchical? Create motivation and interest. Preview material, ask questions, and discuss ideas with friends. The stronger your interest, the greater is your comprehension. Read briskly, but only at a speed that will result in proper comprehension Build a good vocabulary. The best way to improve your vocabulary is to use a dictionary regularly. You might carry around a pocket dictionary and use it to look up new words. Or, you can keep a list of words to look up at the end of the day. Concentrate on roots, prefixes and endings.

CSAT Reading Comprehension

5 © Copyright 2011 Think & Learn Pvt. Ltd Copying or Reproduction without the prior permission of the author is prohibited

TEST-READING COMPREHENSION 01

No of questions-15 Time-30 minutes

Passage 1

Caffeine, the stimulant in coffee, has been called “the most widely used psychoactive substance on Earth.”Synder, Daly and Bruns have recently proposed that caffeine affects behavior by countering the activity in the human brain of a naturally occurring chemical called adenosine. Adenosine normally depresses neuron firing in many areas of the brain. It apparently does this by inhibiting the release of neurotransmitters, chemicals that carry nerve impulses from one neuron to the next. Like many other agents that affect neuron firing, adenosine must first bind to specific receptors on neuronal membranes. There are at least two classes of these receptors, which have been designated A1 and A2. Snyder et al propose that caffeine, which is structurally similar to adenosine, is able to bind to both types of receptors, which prevents adenosine from attaching there and allows the neurons to fire more readily than they otherwise would. For many years, caffeine’s effects have been attributed to its inhibition of the production of phosphodiesterase, an enzyme that breaks down the chemical called cyclic AMP. A number of neurotransmitters exert their effects by first increasing cyclic AMP concentrations in target neurons. Therefore, prolonged periods at the elevated concentrations, as might be brought about by a phosphodiesterase inhibitor, could lead to a greater amount of neuron firing and, consequently, to behavioral stimulation. But Snyder et al point out that the caffeine concentrations needed to inhibit the production of phosphodiesterase in the brain are much higher than those

that produce stimulation. Moreover, other compounds that block phosphodiesterase’s activity are not stimulants. To buttress their case that caffeine acts instead by preventing adenosine binding, Snyder et al compared the stimulatory effects of a series of caffeine derivatives with their ability to dislodge adenosine from its receptors in the brains of mice. “In general,” they reported, “the ability of the compounds to compete at the receptors correlates with their ability to stimulate locomotion in the mouse; i.e., the higher their capacity to bind at the receptors, the higher their ability to stimulate locomotion.” Theophylline, a close structural relative of caffeine and the major stimulant in tea, was one of the most effective compounds in both regards. There were some apparent exceptions to the general correlation observed between adenosine-receptor binding and stimulation. One of these was a compound called 3-isobuty1-1-methylxanthine(IBMX), which bound very well but actually depressed mouse locomotion. Snyder et al suggest that this is not a major stumbling block to their hypothesis. The problem is that the compound has mixed effects in the brain, a not unusual occurrence with psychoactive drugs. Even caffeine, which is generally known only for its stimulatory effects, displays this property, depressing mouse locomotion at very low concentrations and stimulating it at higher ones. 1. The primary purpose of the passage is to (A) discuss a plan for investigation of a phenomenon that is not yet fully understood (B) present two explanations of a phenomenon and reconcile the differences between them (C) summarize two theories and suggest a third theory that overcomes the problems encountered in the first two (D) describe an alternative hypothesis and provide evidence and arguments that support it (E) challenge the validity of a theory by exposing the inconsistencies and contradictions in it 2. According so Snyder et al, caffeine differs from adenosine in that caffeine (A) stimulates behavior in the mouse and in humans, whereas adenosine stimulates behavior in humans only (B) has mixed effects in the brain, whereas adenosine has only a stimulatory effect (C) increases cyclic AMP concentrations in target neurons, whereas adenosine decreases such concentrations (D) permits release of neurotransmitters when it is bound to adenosine receptors, whereas adenosine inhibits such release

CSAT Reading Comprehension

6 © Copyright 2011 Think & Learn Pvt. Ltd Copying or Reproduction without the prior permission of the author is prohibited

(E) inhibits both neuron firing and the production of phosphodiesterase when there is a sufficient concentration in the brain, whereas adenosine inhibits only neuron firing 3. In response to experimental results concerning IBMX, Snyder et al contended that it is not uncommon for psychoactive drugs to have (A) mixed effects in the brain (B) inhibitory effects on enzymes in the brain (C) close structural relationships with caffeine (D) depressive effects on mouse locomotion (E) the ability to dislodge caffeine from receptors in the brain 4. According to Snyder et al, all of the following compounds can bind to specific receptors in the brain EXCEPT (A) IBMX (B) caffeine (C) adenosine (D) theophylline (E) phosphodiesterase 5. Snyder et al suggest that caffeine’s ability to bind to A1 and A2 receptors can be at least partially attributed to which of the following? (A) The chemical relationship between caffeine and phosphodiesterase (B) The structural relationship between caffeine and adenosine (C) The structural similarity between caffeine and neurotransmitters (D) The ability of caffeine to stimulate behavior (E) The natural occurrence of caffeine and adenosine in the brain

Passage 2

Archaeology as a profession faces two major problems. First, it is the poorest of the poor. Only paltry sums are available for excavating and even less is available for publishing the results and preserving the sites once excavated. Yet archaeologists deal with priceless objects every day. Second, there is the problem of illegal excavation, resulting in museum-quality pieces being sold to the highest bidder. I would like to make an outrageous suggestion that would at one stroke provide funds for archaeology and reduce the amount of illegal digging. I would propose that scientific archeological expeditions and governmental authorities sell excavated artifacts on the open market. Such sales would provide substantial funds for the excavation and preservation of archaeological sites and the publication of

results. At the same time, they would break the illegal excavator’s grip on the market, thereby decreasing the inducement to engage in illegal activities. You might object that professionals excavate to acquire knowledge, not money. Moreover, ancient artifacts are part of our global cultural heritage, which should be available for all to appreciate, not sold to the highest bidder. I agree. Sell nothing that has unique artistic merit or scientific value. But, you might reply, everything that comes out of the ground has scientific value. Here we part company. Theoretically, you may be correct in claiming that every artifact has potential scientific value. Practically, you are wrong. I refer to the thousands of pottery vessels and ancient lamps that are essentially duplicates of one another. In one small excavation in Cyprus, archaeologists recently uncovered 2,000 virtually indistinguishable small jugs in a single courtyard, even precious royal seal impressions known as melekh handles have been found in abundance --- more than 4,000 examples so far. The basement of museums are simply not large enough to store the artifacts that are likely to be discovered in the future. There is not enough money even to catalogue the finds; as a result, they cannot be found again and become as inaccessible as if they had never been discovered. Indeed, with the help of a computer, sold artifacts could be more accessible than are the pieces stored in bulging museum basements. Prior to sale, each could be photographed and the list of the purchasers could be maintained on the computer A purchaser could even be required to agree to return the piece if it should become needed for scientific purposes. It would be unrealistic to suggest that illegal digging would stop if artifacts were sold in the open market. But the demand for the clandestine product would be substantially reduced. Who would want an unmarked pot when another was available whose provenance was known, and that was dated stratigraphically by the professional archaeologist who excavated it? 6. The primary purpose of the passage is to propose (A) an alternative to museum display of artifacts (B) a way to curb illegal digging while benefiting the archaeological profession (C) a way to distinguish artifacts with scientific value from those that have no such value (D) the governmental regulation of archaeological sites (E) a new system for cataloguing duplicate artifacts

CSAT Reading Comprehension

7 © Copyright 2011 Think & Learn Pvt. Ltd Copying or Reproduction without the prior permission of the author is prohibited

7. The author implies that all of the following statements about duplicate artifacts are true EXCEPT: (A) A market for such artifacts already exists. (B) Such artifacts seldom have scientific value. (C) There is likely to be a continuing supply of such artifacts. (D) Museums are well supplied with examples of such artifacts. (E) Such artifacts frequently exceed in quality in comparison to those already catalogued in museum collections. 8. Which of the following is mentioned in the passage as a disadvantage of storing artifacts in museum basements? (A) Museum officials rarely allow scholars access to such artifacts. (B) Space that could be better used for display is taken up for storage. (C) Artifacts discovered in one excavation often become separated from each other. (D) Such artifacts are often damaged by variations in temperature and humidity. (E) Such artifacts’ often remain uncatalogued and thus cannot be located once they are put in storage. 9. The author’s argument concerning the effect of the official sale of duplicate artifacts on illegal excavation is based on which of the following assumptions? (A) Prospective purchasers would prefer to buy authenticated artifacts. (B) The price of illegally excavated artifacts would rise. (C) Computers could be used to trace sold artifacts. (D) Illegal excavators would be forced to sell only duplicate artifacts. (E) Money gained from selling authenticated artifacts could be used to investigate and prosecute illegal excavators. 10. The author anticipates which of the following initial objections to the adoption of his proposal? (A) Museum officials will become unwilling to store artifacts. (B) An oversupply of salable artifacts will result and the demand for them will fall. (C) Artifacts that would have been displayed in public places will be sold to private collectors. (D) Illegal excavators will have an even larger supply of artifacts for resale. (E) Counterfeiting of artifacts will become more commonplace.

Passage 3

Federal efforts to aid minority businesses began in the 1960’s when the Small Business Administration (SBA) began making federally guaranteed loans and government-sponsored management and technical assistance available to minority business enterprises. While this program enabled many minority entrepreneurs to form new businesses, the results were disappointing, since managerial inexperience, unfavorable locations, and capital shortages led to high failure rates. Even 15 years after the program was implemented, minority business receipts were not quite two percent of the national economy’s total receipts. Recently federal policymakers have adopted an approach intended to accelerate development of the minority business sector by moving away from directly aiding small minority enterprises and toward supporting larger, growth-oriented minority firms through intermediary companies. In this approach, large corporations participate in the development of successful and stable minority businesses by making use of government sponsored venture capital. The capital is used by a participating company to establish a Minority Enterprise Small Business Investment Company or MESBIC. The MESBIC then provides capital and guidance to minority businesses that have potential to become future suppliers or customers of the sponsoring company. MESBIC’s are the result of the belief that providing established firms with easier access to relevant management techniques and more job-specific experience, as well as substantial amounts of capital, gives those firms a greater opportunity to develop sound business foundations than does simply making general management experience and small amounts of capital available. Further, since potential markets for the minority businesses already exist through the sponsoring companies, the minority businesses face considerably less risk in terms of location and market fluctuation. Following early financial and operating problems, sponsoring corporations began to capitalize MESBIC’s far above the legal minimum of $500,000 in order to generate sufficient income and to sustain the quality of management needed. MESBIC’s are now emerging as increasingly important financing sources for minority enterprises. Ironically, MESBIC staffs, which usually consist of Hispanic and Black professionals, tend to approach investments in minority firms more pragmatically than do many MESBIC directors, who are usually senior managers from sponsoring corporations. The latter often still think mainly in terms of the “social responsibility approach” and thus seem to prefer

CSAT Reading Comprehension

8 © Copyright 2011 Think & Learn Pvt. Ltd Copying or Reproduction without the prior permission of the author is prohibited

deals that are riskier and less attractive than normal investment criteria would warrant. Such differences in viewpoint have produced uneasiness among many minority staff members, who feel that minority entrepreneurs and businesses should be judged by established business considerations. These staff members believe their point of view is closer to the original philosophy of MESBIC’s and they are concerned that, unless a more prudent course is followed, MESBIC directors may revert to policies likely to re-create the disappointing results of the original SBA approach. 11. Which of the following best states the central idea of the passage? (A) The use of MESBIC’s for aiding minority entrepreneurs seems to have greater potential for success than does the original SBA approach. (B) There is a crucial difference in point of view between the staff and directors of some MESBIC’s. (C) After initial problems with management and marketing, minority businesses have begun to expand at a steady rate. (D) Minority entrepreneurs wishing to form new businesses now have several equally successful federal programs on which to rely. (E) For the first time since 1960, large corporations are making significant contributions to the development of minority businesses. 12. According to the passage, the MESBIC approach differs from the SBA approach in that MESBIC’s (A) seek federal contracts to provide markets for minority businesses (B) encourage minority businesses to provide markets for other minority businesses (C) attempt to maintain a specified rate of growth in the minority business sector (D) rely on the participation of large corporations to finance minority businesses (E) select minority businesses on the basis of their location 13. Which of the following does the author cite to support the conclusion that the results of the SBA program were disappointing? (A) The small number of new minority enterprises formed as a result of the program (B) The small number of minority enterprises that took advantage of the management and technical assistance offered under the program (C) The small percentage of the nation’s business receipts earned by minority enterprises following the programs, implementation.

(D) The small percentage of recipient minority enterprises that were able to repay federally guaranteed loans made under the program (E) The small number of minority enterprises that chose to participate in the program 14. Which of the following statements about the SBA program can be inferred from the passage? (A) The maximum term for loans made to recipient businesses was 15 years. (B) Business loans were considered to be more useful to recipient businesses than was management and technical assistance. (C) The anticipated failure rate for recipient businesses was significantly lower than the rate that actually resulted. (D) Recipient businesses were encouraged to relocate to areas more favorable for business development. (E) The capitalization needs of recipient businesses were assessed and then provided for adequately. 15. The author’s primary objective in the passage is to (A) disprove the view that federal efforts to aid minority businesses have been ineffective (B) explain how federal efforts to aid minority businesses have changed since the 1960’s (C) establish a direct link between the federal efforts to aid minority businesses made before the 1960’s and those made in the 1980’s (D) analyze the basis for the belief that job-specific experience is more useful to minority businesses than is general management experience (E) argue that the “social responsibility approach” to aiding minority businesses is superior to any other approach

TEST-READING COMPREHENSION 02

No. of Questions-16 Time- 30 minutes

Passage 1

The majority of successful senior managers do not closely follow the classical rational model of first clarifying goals, assessing the problem, formulating options, estimating likelihoods of success, making a decision, and only then taking action to implement the decision. Rather, in their day-by-day tactical maneuvers, these senior executives rely on what is vaguely termed “intuition” to manage a network of interrelated problems that require them to deal with ambiguity, inconsistency, novelty, and surprise; and to integrate action into the process to thinking.

CSAT Reading Comprehension

9 © Copyright 2011 Think & Learn Pvt. Ltd Copying or Reproduction without the prior permission of the author is prohibited

Generations of writers on management have recognized that some practicing managers rely heavily on intuition. In general, however, such writers display a poor grasp of what intuition is. Some see it as the opposite of rationality; others view it as an excuse for capriciousness. Isenberg’s recent research on the cognitive processes of senior managers reveals that managers’ intuition is neither of these. Rather, senior managers use intuition in at least five distinct ways. First, they intuitively sense when a problem exists. Second, managers rely on intuition to perform well-learned behavior patterns rapidly. This intuition is not arbitrary or irrational, but is based on years of painstaking practice and hands-on experience that build skills. A third function of intuition is to synthesize isolated bits of data and practice into an integrated picture, often in an “Aha!” experience. Fourth, some managers use intuition as a check on the results of more rational analysis. Most senior executives are familiar with the formal decision analysis models and tools, and those who use such systematic methods for reaching decisions are occasionally leery of solutions suggested by these methods which run counter to their sense of the correct course of action. Finally, managers can use intuition to bypass in-depth analysis and move rapidly to engender a plausible solution. Used in this way, intuition is an almost instantaneous cognitive process in which a manager recognizes familiar patterns. One of the implications of the intuitive style of executive management is that “thinking” is inseparable from acting. Since managers often “know” what is right before they can analyze and explain it, they frequently act first and explain later. Analysis is inextricably tied to action in thinking/acting cycles, in which managers develop thoughts about their companies and organizations not by analyzing a problematic situation and then acting, but by acting and analyzing in close concert. Given the great uncertainty of many of the management issues that they face, senior managers often instigate a course of action simply to learn more about an issue. They then use the results of the action to develop a more complete understanding of the issue. One implication of thinking/acting cycles is that action is often part of defining the problem, not just of implementing the solution 1. According to the passage, senior managers use intuition in all of the following ways EXCEPT to (A) speed up of the creation of a solution to a problem (B) identify a problem

(C) bring together disparate facts (D) stipulate clear goals (E) evaluate possible solutions to a problem 2. The passage suggests which of the following about the “writers on management” mentioned in paragraph 2? (A) They have criticized managers for not following the classical rational model of decision analysis. (B) They have not based their analyses on a sufficiently large sample of actual managers. (C) They have relied in drawing their conclusions on what managers say rather than on what managers do. (D) They have misunderstood how managers use intuition in making business decisions. (E) They have not acknowledged the role of intuition in managerial practice. 3. According to the passage, the classical model of decision analysis includes all of the following EXCEPT (A) evaluation of a problem (B) creation of possible solutions to a problem (C) establishment of clear goals to be reached by the decision (D) action undertaken in order to discover more information about a problem (E) comparison of the probable effects of different solutions to a problem 4. It can be inferred from the passage that which of the following would most probably be one major difference in behavior between Manager X, who uses intuition to reach decisions, and Manager Y, who uses only formal decision analysis? (A) Manager X analyzes first and then acts; Manager Y does not. (B) Manager X checks possible solutions to a problem by systematic analysis; Manager Y does not (C) Manager X takes action in order to arrive at the solution to a problem; Manager Y does not. (D) Manager Y draws on years of hands-on experience in creating a solution to a problem; Manager X does not. (E) Manger Y depends on day-to-day tactical maneuvering; manager X does not. 5. The passage provides support for which of the following statements? (A) Managers who rely on intuition are more successful than those who rely on formal decision analysis. (B) Managers cannot justify their intuitive decisions.

CSAT Reading Comprehension

10 © Copyright 2011 Think & Learn Pvt. Ltd Copying or Reproduction without the prior permission of the author is prohibited

(C) Managers’ intuition works contrary to their rational and analytical skills (D) Logical analysis of a problem increases the number of possible solutions. (E) Intuition enables managers to employ their practical experience more efficiently.

Passage 2

Nearly a century ago, biologists found that if they separated an invertebrate animal embryo into two parts at an early stage of its life, it would survive and develop as two normal embryos. This led them to believe that the cells in the early embryo are undetermined in the sense that each cell has the potential to develop in a variety of different ways. Later biologists found that the situation was not so simple. It matters in which plane the embryo is cut. If it is cut in a plane different from the one used by the early investigators, it will not form two whole embryos. A debate arose over what exactly was happening. Which embryo cells are determined, just when do they become irreversibly committed to their fates, and what are the “morphogenetic determinants” that tell a cell what to become? But the debate could not be resolved because no one was able to ask the crucial questions in a form in which they could be pursued productively. Recent discoveries in molecular biology, however, have opened up prospects for a resolution of the debate. Now investigators think they know at least some of the molecules that act as morphogenetic determinants in early development. They have been able to show that, in a sense, cell determination begins even before an egg is fertilized. Studying sea urchins, biologist Paul Gross found that an unfertilized egg contains substances that function as morphogenetic determinants. They are located in the cytoplasm of the egg cell; i.e., in that part of the cell’s protoplasm that lies outside of the nucleus. In the unfertilized egg, the substances are inactive and are not distributed homogeneously. When the egg is fertilized, the substances become active and, presumably, govern the behavior of the genes they interact with. Since the substances are unevenly distributed in the egg, when the fertilized egg divides, the resulting cells are different from the start and so can be qualitatively different in their own gene activity. The substances that Gross studied are maternal messenger RNA’s --products of certain of the maternal genes. He and other biologists studying a wide variety of organisms have found that these particular RNA’s direct, in large part, the

synthesis of histones, a class of proteins that bind to DNA. Once synthesized, the histones move into the cell nucleus, where sections of DNA wrap around them to form a structure that resembles beads, or knots, on a string. The beads are DNA segments wrapped around the histones; the string is the intervening DNA. And it is the structure of these beaded DNA strings that guides the fate of the cells in which they are located. 6. It can be inferred from the passage that the morphogenetic determinants present in the early embryo are (A) located in the nucleus of the embryo cells (B) evenly distributed unless the embryo is not developing normally (C) inactive until the embryo cells become irreversibly committed to their final function (D) identical to those that were already present in the unfertilized egg (E) present in larger quantities than is necessary for the development of a single individual 7. The main topic of the passage is (A) the early development of embryos of lower marine organisms (B) the main contribution of modern embryology to molecular biology (C) the role of molecular biology in disproving older theories of embryonic development (D) cell determination as an issue in the study of embryonic development (E) scientific dogma as a factor in the recent debate over the value of molecular biology 8. According to the passage, when biologists believed that the cells in the early embryo were undetermined, they made which of the following mistakes? (A) They did not attempt to replicate the original experiment of separating an embryo into two parts. (B) They did not realize that there was a connection between the issue of cell determination and the outcome of the separation experiment. (C) They assumed that the results of experiments on embryos did not depend on the particular animal species used for such experiments. (D) They assumed that it was crucial to perform the separation experiment at an early stage in the embryo’s life. (E) They assumed that different ways of separating an embryo into two parts would be equivalent as far as the fate of the two parts was concerned.

CSAT Reading Comprehension

11 © Copyright 2011 Think & Learn Pvt. Ltd Copying or Reproduction without the prior permission of the author is prohibited

9. It can be inferred from the passage that the initial production of histones after an egg is fertilized takes place (A) in the cytoplasm (B) in the maternal genes (C) throughout the protoplasm (D) in the beaded portions of the DNA strings (E) in certain sections of the cell nucleus 10. It can be inferred from the passage that which of the following is dependent on the fertilization of an egg? (A) Copying of maternal genes to produce maternal messenger RNA’s (B) Sythesis of proteins called histones (C) Division of a cell into its nucleus and the cytoplasm (D) Determination of the egg cell’s potential for division (E) Generation of all of a cell’s morphogenetic determinants 11. According to the passage, the morphogenetic determinants present in the unfertilized egg cell are which of the following? (A) Proteins bound to the nucleus (B) Histones (C) Maternal messenger RNA’s (D) Cytoplasm (E) Nonbeaded intervening DNA

Passage 3

In the two decades between 1910 and 1930, over ten percent of the Black population of the United States left the South, where the preponderance of the Black population had been located, and migrated to northern states, with the largest number moving, it is claimed, between 1916 and 1918. It has been frequently assumed, but not proved, that the majority of the migrants in what has come to be called the Great Migration came from rural areas and were motivated by two concurrent factors: the collapse of the cotton industry following the boll weevil infestation, which began in 1898, and increased demand in the North for labor following the cessation of European immigration caused by the outbreak of the First World War in 1914. This assumption has led to the conclusion that the migrants’ subsequent lack of economic mobility in the North is tied to rural background, a background that implies unfamiliarity with urban living and a lack of industrial skills. But the question of who actually left the South has never been rigorously investigated. Although numerous investigations document an exodus from rural southern areas to southern cities prior to the Great Migration, no one has considered whether the same migrants then moved on

to northern cities. In 1910, over 600,000 Black workers, or ten percent of the Black work force, reported themselves to be engaged in “manufacturing and mechanical pursuits,” the federal census category roughly encompassing the entire industrial sector. The Great Migration could easily have been made up entirely of this group and their families. It is perhaps surprising to argue that an employed population could be enticed to move, but an explanation lies in the labor conditions prevalent in the South. About thirty-five percent of the urban Black population in the South was engaged in skilled trades. Some were from the old artisan class of slavery-blacksmiths, masons, carpenters-which had had a monopoly of certain trades, but they were gradually being pushed out by competition, mechanization, and obsolescence. The remaining sixty-five percent, more recently urbanized, worked in newly developed industries---tobacco, lumber, coal and iron manufacture and railroads. Wages in the South, however, were low, and Black workers were aware, through labor recruiters and the Black press, that they could earn more even as unskilled workers in the North than they could as artisans in the South. After the boll weevil infestation, urban Black workers faced competition from the continuing influx of both Black and White rural workers, who were driven to undercut the wages formerly paid for industrial jobs. Thus, a move towards the North would be seen as advantageous to a group that was already urbanized and steadily employed, and the easy conclusion tying their subsequent economic problems in the North to their rural background comes into question. 12. The author indicates explicitly that which of the following records has been a source of information in her investigation? (A) United States Immigration Service reports from 1914 to 1930 (B) Payrolls of southern manufacturing firms between 1910 and 1930 (C) The volume of cotton exports between 1898 and 1910 (D) The federal census of 1910 (E) Advertisements of labor recruiters appearing in southern newspapers after 1910 13. In the passage, the author anticipates which of the following as a possible objection to her argument? (A) It is uncertain how many people actually migrated during the Great Migration.

CSAT Reading Comprehension

12 © Copyright 2011 Think & Learn Pvt. Ltd Copying or Reproduction without the prior permission of the author is prohibited

(B) The eventual economic status of the Great Migration migrants has not been adequately traced. (C) It is not likely that people with steady jobs would have reason to move to another area of the country. (D) It is not true that the term “manufacturing and mechanical pursuits” actually encompasses the entire industrial sector. (E) Of the Black workers living in southern cities, only those in a small number of trades were threatened by obsolescence. 14. According to the passage, which of the following is true about the wages in southern cities in 1910? (A) They were being pushed lower as a result of increased competition. (B) They had begun to rise so that southern industry could attract rural workers. (C) They had increased for skilled workers but decreased for unskilled workers. (D) They had increased in large southern cities but decreased in small southern cities. (E) They had increased in newly developed industries but decreased in the older trades. 15. The author cites each of the following as possible influences in a Black worker’s decision to migrate north in the Great Migration EXCEPT (A) wage levels in northern cities (B) labor recruiters (C) competition from rural workers (D) voting rights in northern states (E) the Black press 16. The primary purpose of the passage is to (A) support an alternative to an accepted methodology (B) present evidence that resolves a contradiction (C) introduce a recently discovered source of information (D) challenge a widely accepted explanation (E) argue that a discarded theory deserves new attention

TEST-READING COMPREHENSION 03

No. of Questions-15 Time- 30 minutes

Passage 1

Fifty feet away three male lions lay by the road. They didn't appear to have a hair on their heads. Noting the color of their noses (leonine noses darken as they age, from pink to black), Craig estimated that they were six years old-young adults. "This is wonderful!" he said, after staring at them for several moments. "This is what we came to see. They really

are maneless." Craig, a professor at the University of Minnesota, is arguably the leading expert on the majestic Serengeti lion, whose head is mantled in long, thick hair. He and Peyton West, a doctoral student who has been working with him in Tanzania, had never seen the Tsavo lions that live some 200 miles east of the Serengeti. The scientists had partly suspected that the maneless males were adolescents mistaken for adults by amateur observers. Now they knew better. The Tsavo research expedition was mostly Peyton's show. She had spent several years in Tanzania, compiling the data she needed to answer a question that ought to have been answered long ago: Why do lions have manes? It's the only cat, wild or domestic, that displays such ornamentation. In Tsavo she was attacking the riddle from the opposite angle. Why do its lions not have manes? (Some "maneless" lions in Tsavo East do have partial manes, but they rarely attain the regal glory of the Serengeti lions.) Does environmental adaptation account for the trait? Are the lions of Tsavo, as some people believe, a distinct subspecies of their Serengeti cousins? The Serengeti lions have been under continuous observation for more than 35 years, beginning with George Schaller's pioneering work in the 1960s. But the lions in Tsavo, Kenya's oldest and largest protected ecosystem, have hardly been studied. Consequently, legends have grown up around them. Not only do they look different, according to the myths, they behave differently, displaying greater cunning and aggressiveness. "Remember too," Kenya: The Rough Guide warns,"Tsavo's lions have a reputation of ferocity." Their fearsome image became well known in 1898, when two males stalled construction of what is now Kenya Railways by allegedly killing and eating 135 Indian and African laborers. A British Army officer in charge of building a railroad bridge over the Tsavo River, Lt. Col. J. H.Patterson, spent nine months pursuing the pair before he brought them to bay and killed them. Stuffed and mounted, they now glare at visitors to the Field Museum in Chicago. Patterson's account of the leonine reign of terror, The Man-Eaters of Tsavo, was an international best seller when published in 1907. Still in print, the book has made Tsavo's lions notorious. That annoys some scientists. "People don't want to give up on mythology," Dennis King told me one day. The zoologist has been working in Tsavo off and on for four years. "I am so sick of this man-eater business. Patterson made a helluva lot of money off that story, but Tsavo's lions are no more likely to turn man-eater than lions from elsewhere."

CSAT Reading Comprehension

13 © Copyright 2011 Think & Learn Pvt. Ltd Copying or Reproduction without the prior permission of the author is prohibited

But tales of their savagery and wiliness don't all come from sensationalist authors looking to make a buck. Tsavo lions are generally larger than lions elsewhere, enabling them to take down the predominant prey animal in Tsavo, the Cape buffalo, one of the strongest, most aggressive animals of Earth. The buffalo don't give up easily: They often kill or severely injure an attacking lion, and a wounded lion might be more likely to turn to cattle and humans for food. And other prey is less abundant in Tsavo than in other traditional lion haunts. A hungry lion is more likely to attack humans. Safari guides and Kenya Wildlife Service rangers tell of lions attacking Land Rovers, raiding camps, stalking tourists. Tsavo is a tough neighborhood, they say, and it breeds tougher lions. But are they really tougher? And if so, is there any connection between their manelessness and their ferocity? An intriguing hypothesis was advanced two years ago by Gnoske and Peterhans: Tsavo lions may be similar to the unmaned cave lions of the Pleistocene. The Serengeti variety is among the most evolved of the species. The latest model, so to speak-while certain morphological differences in Tsavo lions (bigger bodies, smaller skulls, and maybe even lack of a mane) suggest that they are closer to the primitive ancestor of all lions. Craig and Peyton had serious doubts about this idea, but admitted that Tsavo lions pose a mystery to science. 1) The book Man-Eaters of Tsavo annoys some scientists because a) it revealed that Tsavo lions are ferocious b) Patterson made a helluva lot of money from the book by sensationalism. c) it perpetuated the bad name Tsavo lions had. d) it narrated how two male Tsavo lions were killed. 2) The sentence which concludes the first paragraph, "Now they knew better", implies that a) The two scientists were struck by wonder on seeing maneless lions for the first time. b) Though Craig was an expert on the Serengeti lion, now he also knew about the Tsavo lions. c) Craig was now able to confirm that darkening of the noses as lions aged applied to Tsavo lions as well. d) Earlier, Craig and West thought that amateur observers had been mistaken. 3) Which of the following, if true, would weaken the hypothesis advanced by Gnoske and Peterhans most? a) Craig and Peyton develop even more serious doubts about the idea that Tsavo lions are primitive

b) The manelessTsavo East lions are shown to be closer to the cave lions. c) Pleistocene cave lions are shown to be far less violent than believed. d) The morphological variations in body and skull size between the cave and Tsavo lions are found to be insignificant. 4) According to the passage, which of the following has NOT contributed to the popular image of Tsavo lions as savage creatures? a) Tsavo lions have been observed to bring down one of the strongest and most aggressive animals-the Cape buffalo. b) In contrast to the situation in traditional lion haunts, scarcity of non-buffalo prey in the Tsavo makes the Tsavo lions more aggressive c) The Tsavo lion is considered to be less evolved than the Serengeti variety. d) Tsavo lions have been observed to attack vehicles as well as humans.

Passage 2

Recently I spent several hours sitting under a tree in my garden with the social anthropologist William Ury, a Harvard University professor who specializes in the art of negotiation and wrote the bestselling book, “Getting to Yes”. He captivated me with his theory that tribalism protects people from their fear of rapid change. He explained that the pillars of tribalism that humans rely on for security would always counter any significant cultural or social change. In this way, he said, change is never allowed to happen too fast. Technology, for example, is a pillar of society. Ury believes that every time technology moves in a new or radical direction, another pillar such as religion or nationalism will grow stronger -in effect, the traditional and familiar will assume greater importance to compensate for the new and untested. In this manner, human tribes avoid rapid change that leaves people insecure and frightened. But we have all heard that nothing is as permanent as change. Nothing is guaranteed. Pithy expressions, to be sure, but no more than cliches. As Ury says, people don't live that way from day-to-day. On the contrary, they actively seek certainty and stability. They want to know they will be safe. Even so, we scare ourselves constantly with the idea of change. An IBM CEO once said: 'We only re-structure for a good reason, and if we haven't re-structured in a while, that's a good reason.' We are scared that competitors, technology and the consumer will put us out of business -so we have to change all the time just to stay alive. But if we

CSAT Reading Comprehension

14 © Copyright 2011 Think & Learn Pvt. Ltd Copying or Reproduction without the prior permission of the author is prohibited

asked our fathers and grandfathers, would they have said that they lived in a period of little change? Structure may not have changed much. It may just be the speed with which we do things. Change is over-rated, anyway. Consider the automobile. It's an especially valuable example, because the auto industry has spent tens of billions of dollars on research and product development in the last 100 years. Henry Ford's first car had a metal chassis with an internal combustion, gasoline-powered engine, four wheels with rubber tyres, a foot operated clutch assembly and brake system, a steering wheel, and four seats, and it could safely do 18 miles per hour. A hundred years and tens of thousands of research hours later, we drive cars with a metal chassis with an internal combustion, gasoline-powered engine, four wheels with rubber tyres, a foot operated clutch assembly and brake system, a steering wheel, four seats -and the average speed in London in 2001 was 17.5 miles per hour! That's not a hell of a lot of return for the money. Ford evidently doesn't have much to teach us about change. The fact that they're still manufacturing cars is not proof that Ford Motor Co. is a sound organization, just proof that it takes very large companies to make cars in great quantities -making for an almost impregnable entry barrier. Fifty years after the development of the jet engine, planes are also little changed. They've grown bigger, wider and can carry more people. But those are incremental, largely cosmetic changes. Taken together, this lack of real change has come to mean that in travel -whether driving or flying -time and technology have not combined to make things much better. The safety and design have of course accompanied the times and the new volume of cars and flights, but nothing of any significance has changed in the basic assumptions of the final product. At the same time, moving around in cars or aeroplanes becomes less and less efficient all the time. Not only has there been no great change, but also both forms of transport have deteriorated as more people clamour to use them. The same is true for telephones, which took over hundred years to become mobile, or photographic film, which also required an entire century to change. The only explanation for this is anthropological. Once established in calcified organizations, humans do two things: sabotage changes that might render people dispensable, and ensure industry-wide emulation. In the 1960s, German auto companies developed plans to scrap the entire combustion engine for an electrical design. (The same existed in the 1970s in Japan, and in the 1980s in France.) So for 40 years we might have been free of the wasteful and ludicrous dependence on fossil fuels. Why

didn't it go anywhere? Because auto executives understood pistons and carburettors, and would be loath to cannibalize their expertise, along with most of their factories 5) According to the passage, the reason why we continued to be dependent on fossil fuels is that: a) Auto executives did not wish to change. b) No alternative fuels were discovered. c) Change in technology was not easily possible. d) German, Japanese and French companies could not come up with new technologies 6) Which of the following views does the author fully support in the passage? a) Nothing is as permanent as change b) Change is always rapid c) More money spent on innovation leads to more rapid change d) Over decades, structural change has been incremental. 7) According to the passage, which of the following statements is true? a) Executives of automobile companies are inefficient and ludicrous. b) The speed at which an automobile is driven in a city has not changed much in a century c) Anthropological factors have fostered innovation in automobiles by promoting use of new technologies. d) Further innovation in jet engines has been more than incremental. 8) Which of the following best describes one of the main ideas discussed in the passage? a) Rapid change is usually welcomed in society. b) Industry is not as innovative as it is made out to be. c) We should have less change than what we have now. d) Competition spurs companies into radical innovation

Passage 3

Any type of psychology that treats motives, thereby endeavoring to answer the question as to why men behave as they do, is called a dynamic psychology. By its very nature it cannot be merely a descriptive psychology, content to depict the what and the how of human behavior. The boldness of dynamic psychology in striking for causes stands in marked contrast to the timid, “more scientific,” view that seeks nothing else than the establishment of a mathematical function for the relation between some artificially simple stimulus and some equally artificial and simple response. If the psychology of personality is to be

CSAT Reading Comprehension

15 © Copyright 2011 Think & Learn Pvt. Ltd Copying or Reproduction without the prior permission of the author is prohibited

more than a matter of coefficients of correlation it too must be a dynamic psychology, and seek first and foremost a sound and adequate theory of the nature of human dispositions. The type of dynamic psychology almost universally held, though sufficient from the point of view of the abstract motives of the generalized mind, fails to provide a foundation solid enough to bear the weight of any single full-bodied personality. The reason is that prevailing dynamic doctrines refer every mature motive of personality to underlying original instincts, wishes, or needs, shared by all men. Thus, the concert artist’s devotion to his music is sometimes ‘explained’ as an extension of his self-assertive instinct, of the need for sentience, or as a symptom of some repressed striving of the libido. In McDougall’s hormic psychology, for example, it is explicitly stated that only the instincts or propensities can be prime movers. Though capable of extension (on both the receptive and executive sides), they are always few in number, common in all men, and established at birth. The enthusiastic collector of bric-a-brac derives his enthusiasm from the parental instinct; so too does the kindly old philanthropist, as well as the mother of a brood. It does not matter how different these three interests may seem to be, they derive their energy from the same source. The principle is that a very few basic motives suffice for explaining the endless varieties of human interests. The psychoanalyst holds the same over-simplified theory. The number of human interests that he regards as so many canalizations of the one basic sexual instinct is past computation The authors of this type of dynamic psychology are concerning themselves only with mind-in-general. They seek a classification of the common and basic motives by which to explain both normal or neurotic behavior of any individual case. (This is true even though they may regard their own list as heuristic or even as fictional.) The plan really does not work. The very fact that the lists are so different in their composition suggests — what to a naïve observer is plain enough — that motives are almost infinitely varied among men, not only in form but in substance. Not four wishes, nor eighteen propensities, nor any and all combinations of these, even with their extensions and variations, seem adequate to account for the endless variety of goals sought by an endless variety of mortals. Paradoxically enough, in many personalities the few simplified needs or instincts alleged to be the common ground for all motivation, turn out to be completely lacking. The second type of dynamic psychology, the one here

defended, regards adult motives as infinitely varied, and as self-sustaining, contemporary systems, growing out of antecedent systems, but functionally independent of them. Just as a child gradually repudiates his dependence on his parents, develops a will of his own, becomes self-active and self-determining, and outlives his parents, so it is with motives. Each motive has a definite point of origin which may possibly lie in instincts, or, more likely, in the organic tensions of infancy. Chronologically speaking, all adult purposes can be traced back to these seed-forms in infancy, but as the individual matures the tie is broken. Whatever bond remains, is historical, not functional. Such a theory is obviously opposed to psychoanalysis and to all other genetic accounts that assume inflexibility in the root purposes and drives of life. (Freud says that the structure of the Id never changes!) The theory declines to admit that the energies of adult personality are infantile or archaic in nature. Motivation is always contemporary. The life of modern Athens is continuous with the life of the ancient city, but it in no sense depends upon its present “go.” The life of a tree is continuous with that of its seed, but the seed no longer sustains and nourishes the full grown tree. Earlier purposes lead into later purposes, and are abandoned in their favor. William James taught a curious doctrine that has been a matter for incredulous amusement ever since, the doctrine of the transitoriness of instincts. According to this theory — not so quaint as sometimes thought — an instinct appears but once in a lifetime, whereupon it promptly disappears through its transformation into habits. If there are instincts this is no doubt of their fate, for no instinct can retain its motivational force unimpaired after it has been absorbed and recast under the transforming influence of learning. Such is the reasoning of James, and such is the logic of functional autonomy. The psychology of personality must be a psychology of post-instinctive behavior. Woodworth has spoken of the transformation of “mechanisms” into “drives.” A mechanism Woodworth defines as any course of behavior that brings about an adjustment. A drive is any neural process that releases mechanisms especially concerned with consummatory reactions. In the course of learning, many preparatory mechanisms must be developed in order to lead to the consummation of an original purpose. These mechanisms are the effective cause of activity in each succeeding mechanism, furnishing the drive for each stage following in the series. Originally all these mechanisms were merely instrumental, only links in the long chain of processes involved in the achievement of an instinctive purpose; with

CSAT Reading Comprehension

16 © Copyright 2011 Think & Learn Pvt. Ltd Copying or Reproduction without the prior permission of the author is prohibited

time and development, with integration and elaboration, many of these mechanisms become activated directly, setting up a state of desire and tension for activities and objects no longer connected with the original impulse. Activities and objects that earlier in the game were means to an end, now become ends in themselves. Although Woodworth’s choice of quasi-neurological terminology is not the best, his doctrine, or one like it is indispensable in accounting for the infinite number of effective motives possible in human life, and for their severance from the rudimentary desires of infancy. 9) Which is the thematic highlight of this passage? a) The two kinds of dynamic psychology. b) The psychology of post- instinctive behaviour. c) The effective motives in human life. d) The transforming influence of learning 10) Dynamic psychology refers to a) The depiction of human behaviour b) The accepting of why people behave in a certain manner. c) The understanding of human disposition vis-à-vis behaviour. d) The classification of human behaviour. 11) According to the passage, the term “transitoriness of instincts” refers to a) The appearance and transformation of instinct. b) The lingering nature of instincts. c) The lone appearance of instinct. d) The appearance and transformation of instinct.

Passage 4

At the heart of the enormous boom in wine consumption that has taken place in the English-speaking world over the last two decades or so is a fascinating, happy paradox. In the days when wine was exclusively the preserve of a narrow cultural elite, bought either at auctions or from gentleman wine merchants in wing collars and bow-ties, to be stored in rambling cellars and decanted to order by one's butler, the ordinary drinker didn't get a look-in. Wine was considered a highly technical subject, in which anybody without the necessary ability could only fall flat on his or her face in embarrassment. It wasn't just that you needed a refined aesthetic sensibility for the stuff if it wasn't to be hopelessly wasted on you. It required an intimate knowledge of what came from where, and what it was supposed to taste like.

Those were times, however, when wine appreciation essentially meant a familiarity with the great French classics, with perhaps a smattering of other wines—like sherry and port. That was what the wine trade dealt in. These days, wine is bought daily in supermarkets and high-street chains to be consumed that evening, hardly anybody has a cellar to store it in and most don't even possess a decanter. Above all, the wines of literally dozens of countries are available on our market. When a supermarket offers its customers a couple of fruity little numbers from Brazil, we scarcely raise an eyebrow. It seems, in other words, that the commercial jungle that wine has now become has not in the slightest deterred people from plunging adventurously into the thickets in order to taste and see. Consumers are no longer intimidated by the thought of needing to know their Pouilly-Fume from their Pouilly-Fuisse, just at the very moment when there is more to know than ever before. The reason for this new mood of confidence is not hard to find. It is on every wine label from Australia,New Zealand, South Africa and the United States: the name of the grape from which the wine is made. At one time that might have sounded like a fairly technical approach in itself. Why should native English speakers know what Cabernet Sauvignon or Chardonnay were? The answer lies in the popularity that wines made from those grape varieties now enjoy. Consumers effectively recognize them as brand names, and have acquired a basic lexicon of wine that can serve them even when confronted with those Brazilian upstarts. In the wine heartlands of France, they are scared to death of that trend—not because they think their wine isn't as good as the best from California or South Australia (what French winemaker will ever admit that?) but because they don't traditionally call their wines Cabernet Sauvignon or Chardonnay. They call them Chateau Ducru-Beaucaillou or Corton-Charlemagne, and they aren't about to change. Some areas, in the middle of southern France, have now produced a generation of growers using the varietal names on their labels and are tempting consumers back to French wine. It will be an uphill struggle, but there is probably no other way if France is to avoid simply becoming a specialty source of old-fashioned wines for old fashioned connoisseurs. Wine consumption was also given a significant boost in the early 1990s by the work of Dr. Serge Renaud, who has spent many years investigating the reasons for the uncannily low incidence of coronary heart disease in the south of France. One of his major findings is that the fat-derived cholesterol that builds up in the arteries and can eventually lead to

CSAT Reading Comprehension

17 © Copyright 2011 Think & Learn Pvt. Ltd Copying or Reproduction without the prior permission of the author is prohibited

heart trouble, can be dispersed by the tannins in wine. Tannin is derived from the skins of grapes, and is therefore present in higher levels in red wines, because they have to be infused with their skins to attain the red colour. That news caused a huge upsurge in red wine consumption in the United States. It has not been accorded the prominence it deserves in the UK, largely because the medical profession still sees all alcohol as a menace to health, and is constantly calling for it to be made prohibitively expensive. Certainly, the manufacturers of anticoagulant drugs might have something to lose if we all got the message that we would do just as well by our hearts by taking half a bottle of red wine every day! 12) Which one of the following CANNOT be reasonably attributed to the labelling strategy followed by wine producers in English-speaking countries? a) Consumers buy wines on the basis of their familiarity with a grape variety's name b) Even ordinary customers now have more access to technical knowledge about wine c) Consumers are able to appreciate better quality wines d) Some non-English speaking countries like Brazil indicate grape variety names on their labels. 13) What according to the author should the French do to avoid becoming a producer of merely old fashioned wines? a) Follow the labelling strategy of the English-speaking countries b) Give their wines English names c) Introduce fruity wines as Brazil has done. d) Produce the wines that have become popular in the English-speaking world. 14) Which one of the following, if true, would provide most support for Dr. Renaud's findings about the effect of tannins? a) A survey showed that film celebrities based in France have a low incidence of coronary heart disease. b) Measurements carried out in southern France showed red wine drinkers had significantly higher levels of coronary heart incidence than white wine drinkers did. c) Data showed a positive association between sales of red wine and incidence of coronary heart disease. d) Long-term surveys in southern France showed that the incidence of coronary heart disease was significantly lower in red wine drinkers than in those who did not drink red wine 15) The development which has created fear among winemakers in the wine heartlands of France is the

a) tendency not to name wines after the grape varieties that are used in the wines. b) 'education' that consumers have derived from wine labels from English-speaking countries c) new generation of local winegrowers who use labels that show names of grape varieties. d) ability of consumers to understand a wine's qualities when confronted with "Brazilian

TEST-READING COMPREHENSION 04

No. of Questions-15 Time- 30 minutes

Passage 1

In 1896 a Georgia couple suing for damages in the accidental death of their two year old was told that since the child had made no real economic contribution to the family, there was no liability for damages. In contrast, less than a century later, in 1979, the parents of a three year old sued in New York for accidental-death damages and won an award of $750,000. The transformation in social values implicit in juxtaposing these two incidents is the subject of Viviana Zelizer’s excellent book, Pricing the Priceless Child. During the nineteenth century, she argues, the concept of the “useful” child who contributed to the family economy gave way gradually to the present-day notion of the “useless” child who, though producing no income for, and indeed extremely costly to, its parents, is yet considered emotionally “priceless.” Well established among segments of the middle and upper classes by the mid-1800’s, this new view of childhood spread throughout society in the late-nineteenth and early-twentieth centuries as reformers introduced child-labor regulations and compulsory education laws predicated in part on the assumption that a child’s emotional value made child labor taboo. For Zelizer, the origins of this transformation were many and complex. The gradual erosion of children’s productive value in a maturing industrial economy, the decline in birth and death rates, especially in child mortality, and the development of the companionate family (a family in which members were united by explicit bonds of love rather than duty) were all factors critical in changing the assessment of children’s worth. Yet “expulsion of children from the ‘cash nexus,’...although clearly shaped by profound changes in the economic, occupational, and family structures,” Zelizer maintains. “was also part of a cultural process ‘of sacralization’ of children’s lives. ” Protecting children from the crass business world became enormously important for late-

CSAT Reading Comprehension

18 © Copyright 2011 Think & Learn Pvt. Ltd Copying or Reproduction without the prior permission of the author is prohibited

nineteenth-century middle-class Americans, she suggests; this sacralization was a way of resisting what they perceived as the relentless corruption of human values by the marketplace. In stressing the cultural determinants of a child’s worth, Zelizer takes issue with practitioners of the new “sociological economics,” who have analyzed such traditionally sociological topics as crime, marriage, education, and health solely in terms of their economic determinants. Allowing only a small role for cultural forces in the form of individual “preferences,” these sociologists tend to view all human behavior as directed primarily by the principle of maximizing economic gain. Zelizer is highly critical of this approach, and emphasizes instead the opposite phenomenon: the power of social values to transform price. As children became more valuable in emotional terms, she argues, their “exchange” or “surrender” value on the market, that is, the conversion of their intangible worth into cash terms, became much greater. 1. It can be inferred from the passage that accidental-death damage awards in America during the nineteenth century tended to be based principally on the (A) earnings of the person at time of death (B) wealth of the party causing the death (C) degree of culpability of the party causing the death (D) amount of money that had been spent on the person killed (E) amount of suffering endured by the family of the person killed 2. It can be inferred from the passage that in the early 1800’s children were generally regarded by their families as individuals who (A) needed enormous amounts of security and affection (B) required constant supervision while working (C) were important to the economic well-being of a family (D) were unsuited to spending long hours in school (E) were financial burdens assumed for the good of society 3. Which of the following alternative explanations of the change in the cash value of children would be most likely to be put forward by sociological economists as they are described in the passage? (A) The cash value of children rose during the nineteenth century because parents began to increase their emotional investment in the upbringing of their children. (B) The cash value of children rose during the nineteenth century because their expected earnings over the course of a lifetime increased greatly.

(C) The cash value of children rose during the nineteenth century because the spread of humanitarian ideals resulted in a wholesale reappraisal of the worth of an individual (D) The cash value of children rose during the nineteenth century because compulsory education laws reduced the supply, and thus raised the costs, of available child labor. (E)The cash value of children rose during the nineteenth century because of changes in the way negligence law assessed damages in accidental-death cases. 4. The primary purpose of the passage is to (A) review the literature in a new academic subfield (B) present the central thesis of a recent book (C) contrast two approaches to analyzing historical change (D) refute a traditional explanation of a social phenomenon (E) encourage further work on a neglected historical topic 5. It can be inferred from the passage that which of the following statements was true of American families over the course of the nineteenth century? (A) The average size of families grew considerably (B) The percentage of families involved in industrial work declined dramatically. (C) Family members became more emotionally bonded to one another. (D) Family members spent an increasing amount of time working with each other. (E) Family members became more economically dependent on each other. 6. Zelizer refers to all of the following as important influences in changing the assessment of children’s worth EXCEPT changes in (A) the mortality rate (B) the nature of industry (C) the nature of the family (D) attitudes toward reform movements (E) attitudes toward the marketplace

Passage 2

Prior to 1975, union efforts to organize public-sector clerical workers, most of whom are women, were somewhat limited. The factors favoring unionization drives seem to have been either the presence of large numbers of workers, as in New York City, to make it worth the effort, or the concentration of small numbers in one or two locations, such as a hospital, to make it relatively easy, Receptivity to unionization on the workers, part was also a consideration, but when there were large numbers involved or the clerical workers were the only unorganized group in a jurisdiction,

CSAT Reading Comprehension

19 © Copyright 2011 Think & Learn Pvt. Ltd Copying or Reproduction without the prior permission of the author is prohibited

the multioccupational unions would often try to organize them regardless of the workers’ initial receptivity. The strategic reasoning was based, first, on the concern that politicians and administrators might play off unionized against nonunionized workers, and, second, on the conviction that a fully unionized public work force meant power, both at the bargaining table and in the legislature. In localities where clerical workers were few in number, were scattered in several workplaces, and expressed no interest in being organized, unions more often than not ignored them in the pre-1975 period. But since the mid-1970’s, a different strategy has emerged. In 1977, 34 percent of government clerical workers were represented by a labor organization, compared with 46 percent of government professionals, 44 percent of government blue-collar workers, and 41 percent of government service workers, Since then, however, the biggest increases in public-sector unionization have been among clerical workers. Between 1977 and 1980, the number of unionized government workers in blue-collar and service occupations increased only about 1.5 percent, while in the white-collar occupations the increase was 20 percent and among clerical workers in

particular, the increase was 22 percent.

What accounts for this upsurge in unionization among clerical workers? First, more women have entered the work force in the past few years, and more of them plan to remain working until retirement age. Consequently, they are probably more concerned than their predecessors were about job security and economic benefits. Also, the women’s movement has succeeded in legitimizing the economic and political activism of women on their own behalf, thereby producing a more positive attitude toward unions. The absence of any comparable increase in unionization among private-sector clerical workers, however, identifies the primary catalyst-the structural change in the multioccupational public-sector unions themselves. Over the past twenty years, the occupational distribution in these unions has been steadily shifting from predominantly blue-collar to predominantly white-collar. Because there are far more women in white-collar jobs, an increase in the proportion of female members has accompanied the occupational shift and has altered union policy-making in favor of organizing women and addressing women’s issues.

7) According to the passage, the public-sector workers who

were most likely to belong to unions in 1977 were

(A) professionals (B) managers (C) clerical workers (D) service workers (E) blue-collar workers

8) The author cites union efforts to achieve a fully unionized

work force in order to account for why (A) politicians might try to oppose public-sector union organizing (B) public-sector unions have recently focused on organizing women (C) early organizing efforts often focused on areas where there were large numbers of workers (D) union efforts with regard to public-sector clerical workers increased dramatically after 1975 (E) unions sometimes tried to organize workers regardless of the workers’ initial interest in unionization

9) The author’s claim that, since the mid-1970’s, a new

strategy has emerged in the unionization of public-sector

clerical workers would be strengthened if the author (A) described more fully the attitudes of clerical workers toward labor unions (B) compared the organizing strategies employed by private-sector unions with those of public-sector unions (C) explained why politicians and administrators sometimes oppose unionization of clerical workers (D) indicated that the number of unionized public-sector clerical workers was increasing even before the mid-1970’s (E) showed that the factors that favored unionization drives among these workers prior to 1975 have decreased in importance 10) According to the passage, in the period prior to 1975, each of the following considerations helped determine whether a union would attempt to organize a certain group of clerical workers EXCEPT (A) the number of clerical workers in that group (B) the number of women among the clerical workers in that group (C) whether the clerical workers in that area were concentrated in one workplace or scattered over several workplaces (D) the degree to which the clerical workers in that group were interested in unionization (E) whether all the other workers in the same jurisdiction as that group of clerical workers were unionized

CSAT Reading Comprehension

20 © Copyright 2011 Think & Learn Pvt. Ltd Copying or Reproduction without the prior permission of the author is prohibited

11) The author states that which of the following is a consequence of the women’s movement of recent years? (A) An increase in the number of women entering the work force (B) A structural change in multioccupational public-sector unions (C) A more positive attitude on the part of women toward unions (D) An increase in the proportion of clerical workers that are women (E) An increase in the number of women in administrative positions

12) The main concern of the passage is to (A) advocate particular strategies for future efforts to organize certain workers into labor unions (B) explain differences in the unionized proportions of various groups of public-sector workers (C) evaluate the effectiveness of certain kinds of labor unions that represent public-sector workers (D) analyzed and explain an increase in unionization among a certain category of workers (E) describe and distinguish strategies appropriate to organizing different categories of workers

TEST-READING COMPREHENSION 05

No. of Questions-15 Time- 30 minutes

Passage 1

Many United States companies have, unfortunately, made the search for legal protection from import competition into a major line of work. Since 1980 the United States International Trade Commission (ITC) has received about 280 complaints alleging damage from imports that benefit from subsidies by foreign governments. Another 340 charge that foreign companies “dumped” their products in the United States at “less than fair value.” Even when no unfair practices are alleged, the simple claim that an industry has been injured by imports is sufficient grounds to seek relief. Contrary to the general impression, this quest for import relief has hurt more companies than it has helped. As corporations begin to function globally, they develop an intricate web of marketing, production, and research relationships, The complexity of these relationships makes it unlikely that a system of import relief laws will meet the strategic needs of all the units under the same parent company.

Internationalization increases the danger that foreign companies will use import relief laws against the very companies the laws were designed to protect. Suppose a United States-owned company establishes an overseas plant to manufacture a product while its competitor makes the same product in the United States. If the competitor can prove injury from the imports---and that the United States company received a subsidy from a foreign government to build its plant abroad—the United States company’s products will be uncompetitive in the United States, since they would be subject to duties. Perhaps the most brazen case occurred when the ITC investigated allegations that Canadian companies were injuring the United States salt industry by dumping rock salt, used to de-ice roads. The bizarre aspect of the complaint was that a foreign conglomerate with United States operations was crying for help against a United States company with foreign operations. The “United States” company claiming injury was a subsidiary of a Dutch conglomerate, while the “Canadian” companies included a subsidiary of a Chicago firm that was the second-largest domestic producer of rock salt. 1. The passage is chiefly concerned with (A) arguing against the increased internationalization of United States corporations (B) warning that the application of laws affecting trade frequently has unintended consequences (C) demonstrating that foreign-based firms receive more subsidies from their governments than United States firms receive from the United States government (D) advocating the use of trade restrictions for “dumped” products but not for other imports (E) recommending a uniform method for handling claims of unfair trade practices 2. It can be inferred from the passage that the minimal basis for a complaint to the International Trade Commission is which of the following? (A) A foreign competitor has received a subsidy from a foreign government. (B) A foreign competitor has substantially increased the volume of products shipped to the United States. (C) A foreign competitor is selling products in the United States at less than fair market value. (D) The company requesting import relief has been injured by the sale of imports in the United States. (E) The company requesting import relief has been barred from exporting products to the country of its foreign competitor.

CSAT Reading Comprehension

21 © Copyright 2011 Think & Learn Pvt. Ltd Copying or Reproduction without the prior permission of the author is prohibited

3. The last paragraph performs which of the following functions in the passage? (A) It summarizes the discussion thus far and suggests additional areas of research. (B) It presents a recommendation based on the evidence presented earlier. (C) It discusses an exceptional case in which the results expected by the author of the passage were not obtained. (D) It introduces an additional area of concern not mentioned earlier. (E) It cites a specific case that illustrates a problem presented more generally in the previous paragraph. 4. The passage warns of which of the following dangers? (A) Companies in the United States may receive no protection from imports unless they actively seek protection from import competition. (B) Companies that seek legal protection from import competition may incur legal costs that far exceed any possible gain. (C) Companies that are United States-owned but operate internationally may not be eligible for protection from import competition under the laws of the countries in which their plants operate. (D) Companies that are not United States-owned may seek legal protection from import competition under United States import relief laws. (E) Companies in the United States that import raw materials may have to pay duties on those materials. 5. The passage suggests that which of the following is most likely to be true of United States trade laws? (A) They will eliminate the practice of “dumping” products in the United States. (B) They will enable manufacturers in the United States to compete more profitably outside the United States. (C) They will affect United States trade with Canada more negatively than trade with other nations. (D) Those that help one unit within a parent company will not necessarily help other units in the company. (E) Those that are applied to international companies will accomplish their intended result. 6. It can be inferred from the passage that the author believes which of the following about the complaint mentioned in the last paragraph? (A) The ITC acted unfairly toward the complainant in its investigation. (B) The complaint violated the intent of import relief laws.

(C) The response of the ITC to the complaint provided suitable relief from unfair trade practices to the complainant. (D) The ITC did not have access to appropriate information concerning the case. (E) Each of the companies involved in the complaint acted in its own best interest.

Passage 2

At the end of the nineteenth century, a rising interest in Native American customs and an increasing desire to understand Native American culture prompted ethnologists to begin recording the life stories of Native American. Ethnologists had a distinct reason for wanting to hear the stories: they were after linguistic or anthropological data that would supplement their own field observations, and they believed that the personal stories, even of a single individual, could increase their understanding of the cultures that they had been observing from without. In addition many ethnologists at the turn of the century believed that Native American manners and customs were rapidly disappearing, and that it was important to preserve for posterity as much information as could be adequately recorded before the cultures disappeared forever. There were, however, arguments against this method as a way of acquiring accurate and complete information. Franz Boas, for example, described autobiographies as being “of limited value, and useful chiefly for the study of the perversion of truth by memory,” while Paul Radin contended that investigators rarely spent enough time with the tribes they were observing, and inevitably derived results too tinged by the investigator’s own emotional tone to be reliable. Even more importantly, as these life stories moved from the traditional oral mode to recorded written form, much was inevitably lost. Editors often decided what elements were significant to the field research on a given tribe. Native Americans recognized that the essence of their lives could not be communicated in English and that events that they thought significant were often deemed unimportant by their interviewers. Indeed, the very act of telling their stories could force Native American narrators to distort their cultures, as taboos had to be broken to speak the names of dead relatives crucial to their family stories. Despite all of this, autobiography remains a useful tool for ethnological research: such personal reminiscences and impressions, incomplete as they may be, are likely to throw more light on the working of the mind and emotions than

CSAT Reading Comprehension

22 © Copyright 2011 Think & Learn Pvt. Ltd Copying or Reproduction without the prior permission of the author is prohibited

any amount of speculation from an ethnologist or ethnological theorist from another culture. 7. Which of the following best describes the organization of the passage? (A) The historical backgrounds of two currently used research methods are chronicled. (B) The validity of the data collected by using two different research methods is compared. (C) The usefulness of a research method is questioned and then a new method is proposed. (D) The use of a research method is described and the limitations of the results obtained are discussed. (E) A research method is evaluated and the changes necessary for its adaptation to other subject areas are discussed. 8. Which of the following is most similar to the actions of nineteenth-century ethnologists in their editing of the life stories of Native Americans? (A) A witness in a jury trial invokes the Fifth Amendment in order to avoid relating personally incriminating evidence. (B) A stockbroker refuses to divulge the source of her information on the possible future increase in a stock’s value. (C) A sports announcer describes the action in a team sport with which he is unfamiliar. (D) A chef purposely excludes the special ingredient from the recipe of his prizewinning dessert. (E) A politician fails to mention in a campaign speech the similarities in the positions held by her opponent for political office and by herself. 9. According to the passage, collecting life stories can be a useful methodology because (A) life stories provide deeper insights into a culture than the hypothesizing of academics who are not members of that culture (B) life stories can be collected easily and they are not subject to invalid interpretations (C) ethnologists have a limited number of research methods from which to choose (D) life stories make it easy to distinguish between the important and unimportant features of a culture (E) the collection of life stories does not require a culturally knowledgeable investigator 10. Information in the passage suggests that which of the following may be a possible way to eliminate bias in the editing of life stories?

(A) Basing all inferences made about the culture on an ethnological theory (B) Eliminating all of the emotion-laden information reported by the informant (C) Translating the informant’s words into the researcher’s language (D) Reducing the number of questions and carefully specifying the content of the questions that the investigator can ask the informant (E) Reporting all of the information that the informant provides regardless of the investigator’s personal opinion about its intrinsic value 11. The primary purpose of the passage as a whole is to (A) question an explanation (B) correct a misconception (C) critique a methodology (D) discredit an idea (E) clarify an ambiguity 12. It can be inferred from the passage that a characteristic of the ethnological research on Native Americans conducted during the nineteenth century was the use of which of the following? (A) Investigators familiar with the culture under study (B) A language other than the informant’s for recording life stories (C) Life stories as the ethnologist’s primary source of information (D) Complete transcriptions of informants’ descriptions of tribal beliefs (E) Stringent guidelines for the preservation of cultural data Passage 3- Poem Let us go then, you and I, When the evening is spread out against the sky Like a patient etherized upon a table; Let us go, through certain half-deserted streets, The muttering retreats Of restless nights in one-night cheap hotels And sawdust restaurants with oyster-shells: Streets that follow like a tedious argument Of insidious intent To lead you to an overwhelming question. . . Oh, do not ask, “What is it?” Let us go and make our visit. In the room the women come and go Talking of Michelangelo. The yellow fog that rubs its back upon the window-panes

CSAT Reading Comprehension

23 © Copyright 2011 Think & Learn Pvt. Ltd Copying or Reproduction without the prior permission of the author is prohibited

The yellow smoke that rubs its muzzle on the window-panes Licked its tongue into the corners of the evening Lingered upon the pools that stand in drains, Let fall upon its back the soot that falls from chimneys, Slipped by the terrace, made a sudden leap, And seeing that it was a soft October night Curled once about the house, and fell asleep. And indeed there will be time For the yellow smoke that slides along the street, Rubbing its back upon the window-panes; There will be time, there will be time To prepare a face to meet the faces that you meet; There will be time to murder and create, And time for all the works and days of hands That lift and drop a question on your plate; Time for you and time for me, And time yet for a hundred indecisions And for a hundred visions and revisions Before the taking of a toast and tea. In the room the women come and go Talking of Michelangelo. And indeed there will be time To wonder, “Do I dare?” and, “Do I dare?” Time to turn back and descend the stair, With a bald spot in the middle of my hair— *They will say: “How his hair is growing thin!”+ My morning coat, my collar mounting firmly to the chin, My necktie rich and modest, but asserted by a simple pin— *They will say: “But how his arms and legs are thin!”+ Do I dare Disturb the universe? In a minute there is time for decisions and revisions which a minute will reverse. For I have known them all already, known them all; Have known the evenings, mornings, afternoons, I have measured out my life with coffee spoons; I know the voices dying with a dying fall Beneath the music from a farther room. So how should I presume?

13) Which of the following meanings can be inferred from the lines “o I dare Disturb the universe?” (a) The author is referring to his bright future. (b) The author fears that he will cause some major upheaval in world. (c) The author refers to the ‘status quo’ in which he is in. (d) The author expresses his feeling of being pinned against a

wall. (e) The author is apprehensive about his last days.

14) What, according to the passage, is the reason for the author’s optimism? (a) That the women are talking of Michelangelo. (b) That the yellow fog rubs upon the window-panes. (c) That it was an October night. (d) That there will be moments for everything. (e) That the falling soot made a sudden leap.

15) In the first ten lines of the passage the author embodies which of the following with human attributes? (a) toast (b) restaurants (c) intent (d) retreats (e) arguments 16) In the passage, evening is compared to: (a) The spreading sky (b) The anesthetized patient (c) Wicked people (d) The deserted streets (e) A walk in the streets

TEST-READING COMPREHENSION 06

No. of Questions-15 Time- 30 minutes

Passage 1 All of the cells in a particular plant start out with the same complement of genes. How then can these cells differentiate and form structures as different as roots, stems, leaves, and fruits? The answer is that only a small subset of the genes in a particular kind of cell are expressed, or turned on, at a given time. This is accomplished by a complex system of chemical messengers that in plants include hormones and other regulatory molecules. Five major hormones have been identified: auxin, abscisic acid, cytokinin, ethylene, and gibberellin. Studies of plants have now identified a new class of regulatory molecules called oligosaccharins. Unlike the oligosaccharins, the five well-known plant hormones are pleiotropic rather than specific, that is, each has more than one effect on the growth and development of plants. They have so many simultaneous effects that they

CSAT Reading Comprehension

24 © Copyright 2011 Think & Learn Pvt. Ltd Copying or Reproduction without the prior permission of the author is prohibited

are not very useful in artificially controlling the growth of crops. Auxin, for instance, stimulates the rate of cell elongation, causes shoots to grow up and roots to grow down, and inhibits the growth of lateral shoots. Auxin also causes the plant to develop a vascular system, to form lateral roots, and to produce ethylene. The pleiotropy of the five well-studied plant hormones is somewhat analogous to that of certain hormones in animal. For example, hormones from the hypothalamus in the brain stimulate the anterior lobe of the pituitary gland to synthesize and release many different hormones, one of which stimulates the release of hormones from the adrenal cortex. These hormones have specific effects on target organs all over the body. One hormone stimulates the thyroid gland, for example, another the ovarian follicle cells, and so forth. In other words, there is a hierarchy of hormones. Such a hierarchy may also exist in plants. Oligosaccharins are fragments of the cell wall released by enzymes: different enzymes release different oligosaccharins. There are indications that pleiotropic plant hormones may actually function by activating the enzymes that release these other, more specific chemical messengers from the cell wall. 1. According to the passage, the five well-known plant hormones are not useful in controlling the growth of crops because (A) It is not known exactly what functions the hormones perform (B) Each hormone has various effects on plants (C) None of the hormones can function without the others (D) Each hormone has different effects on different kinds of plants (E) Each hormone works on only a small subset of a cell’s genes at any particular time 2. The passage suggests that the place of hypothalamic hormones in the hormonal hierarchies of animals is similar to the place of which of the following in plants? (A) Plant cell walls (B) The complement of genes in each plant cell (C) A subset of a plant cell’s gene complement (D) The five major hormones (E) The oligosaccharins 3. The passage suggests that which of the following is a function likely to be performed by an oligosaccharin? (A) To stimulate a particular plant cell to become part of a plant’s root system

(B) To stimulate the walls of a particular cell to produce other oligosaccharins (C) To activate enzymes that release specific chemical messengers from plant cell walls (D) To duplicate the gene complement in a particular plant cell (E) To produce multiple effects on a particular subsystem of plant cells 4. The author mentions specific effects that auxin has on plant development in order to illustrate the (A) Point that some of the effects of plant hormones can be harmful (B) Way in which hormones are produced by plants (C) Hierarchical nature of the functioning of plant hormones (D) Differences among the best-known plant hormones (E) Concept of pleiotropy as it is exhibited by plant hormones 5. According to the passage, which of the following best describes a function performed by oligosaccharins? (A) Regulating the daily functioning of a plant’s cells (B) Interacting with one another to produce different chemicals (C) Releasing specific chemical messengers from a plant’s cell walls (D) Producing the hormones that cause plant cells to differentiate to perform different functions (E) Influencing the development of a plant’s cells by controlling the expression of the cells’ genes 6. The passage suggests that, unlike the pleiotropic hormones, oligosaccharins could be used effectively to (A) Trace the passage of chemicals through the walls of cells (B) Pinpoint functions of other plant hormones (C) Artificially control specific aspects of the development of crops (D) Alter the complement of genes in the cells of plants (E) Alter the effects of the five major hormones on plant development

Passage 2

Two recent publications offer different assessment of the career of the famous British nurse Florence Nightingale. A book by Anne Summers seeks to debunk the idealizations and present a reality at odds with Nightingale’s heroic reputation. According to Summers, Nightingale’s importance during the Crimean War has been exaggerated: not until near the war’s end did she become supervisor of the female nurses. Additionally, Summers writes that the contribution of the nurses to the relief of the wounded was at best marginal. The prevailing problems of military

CSAT Reading Comprehension

25 © Copyright 2011 Think & Learn Pvt. Ltd Copying or Reproduction without the prior permission of the author is prohibited

medicine were caused by army organizational practices, and the addition of a few nurses to the medical staff could be no more than symbolic. Nightingale’s place in the national pantheon, Summers asserts, is largely due to the propagandistic efforts of contemporary newspaper reporters. By contrast, the editors of a new volume of Nightingale’s letters view Nightingale as a person who Significantly influenced not only her own age but also subsequent generations. They highlight her ongoing efforts to reform sanitary conditions after the war. For example, when she learned that peacetime living conditions in British barracks were so horrible that the death rate of enlisted men far exceeded that of neighboring civilian populations, she succeeded in persuading the government to establish a Royal Commission on the Health of the Army. She used sums raised through public contributions to found a nurses’ training hospital in London. Even in administrative matters, the editors assert, her practical intelligence was formidable: as recently as 1947 the British Army’s medical services were still using the cost-accounting system she had devised in the 1860’s. I believe that the evidence of her letters supports continued respect for Nightingale’s brilliance and creativity. When counseling a village schoolmaster to encourage children to use their faculties of observation, she sounds like a modern educator. Her insistence on classifying the problems of the needy in order to devise appropriate treatments is similar to the approach of modern social workers. In sum, although Nightingale may not have achieved all of her goals during the Crimean War, her breadth of vision and ability to realize ambitious projects have earned her an eminent place among the ranks of social pioneers. 7. The passage is primarily concerned with evaluating (A) The importance of Florence Nightingale’s innovations in the field of nursing (B) Contrasting approaches to the writing of historical biography (C) Contradictory accounts of Florence Nightingale’s historical significance (D) The quality of health care in nineteenth-century England (E) The effect of the Crimean War on developments in the field of health care 8. According to the passage, the editors of Nightingale’s letters credit her with contributing to which of the following? (A) Improving of the survival rate for soldiers in British Army hospitals during the Crimean War

(B) The development of a nurses’ training curriculum that was far in advance of its day (C) The increase in the number of women doctors practicing in British Army hospitals (D) Establishment of the first facility for training nurses at a major British university (E) The creation of an organization for monitoring the peacetime living conditions of British soldiers 9. The passage suggests which of the following about Nightingale’s relationship with the British public of her day? (A) She was highly respected, her projects receiving popular and governmental support. (B) She encountered resistance both from the army establishment and the general public. (C) She was supported by the working classes and opposed by the wealthier classes. (D) She was supported by the military establishment but had to fight the governmental bureaucracy. (E) After initially being received with enthusiasm, she was quickly forgotten. 10. The passage suggests which of the following about sanitary conditions in Britain after the Crimean War? (A) While not ideal, they were superior to those in other parts of the world. (B) Compared with conditions before the war, they had deteriorated. (C) They were more advanced in rural areas than in the urban centers. (D) They were worse in military camps than in the neighboring civilian populations. (E) They were uniformly crude and unsatisfactory throughout England. 11. Which of the following statements regarding the differing interpretations of Nightingale’s importance would the author most likely agree? (A) Summers misunderstood both the importance of Nightingale’s achievements during the Crimean War and her subsequent influence on British policy. (B) The editors of Nightingale’s letters made some valid points about her practical achievements, but they still exaggerated her influence on subsequent generations. (C) Although Summers’ account of Nightingale’s role in the Crimean War may be accurate, she ignored evidence of Nightingales’ subsequent achievement that suggests that her reputation as an eminent social reformer is well deserved. (D) The editors of Nightingale’s letters mistakenly propagated the outdated idealization of Nightingale that

CSAT Reading Comprehension

26 © Copyright 2011 Think & Learn Pvt. Ltd Copying or Reproduction without the prior permission of the author is prohibited

only impedes attempts to arrive at a balance assessment of her true role. (E) The evidence of Nightingale’s letters supports Summers’ conclusions both about Nightingale’s activities and about her influence. 12. Which of the following is an assumption underlying the author’s assessment of Nightingale’s creativity? (A) Educational philosophy in Nightingale’s day did not normally emphasize developing children’s ability to observe. (B) Nightingale was the first to notice the poor living conditions in British military barracks in peacetime. (C) No educator before Nightingale had thought to enlist the help of village schoolmasters in introducing new teaching techniques. (D) Until Nightingale began her work, there was no concept of organized help for the needy in nineteenth-century Britain. (E) The British Army’s medical services had no cost-accounting system until Nightingale devised one in the 1860’s. 13. In the last paragraph, the author is primarily concerned with (A) Summarizing the arguments about Nightingale presented in the first two paragraphs (B) Refuting the view of Nightingale’s career presented in the preceding paragraph (C) Analyzing the weaknesses of the evidence presented elsewhere in the passage (D) Citing evidence to support a view of Nightingale’s career (E) Correcting a factual error occurring in one of the works under review

TEST-READING COMPREHENSION 07

No. of Questions-14 Time- 30 minutes

PASSAGE – 1

Theresa Kelley and Thomas Pfau rehearse a debate- I would call it an anxiety-about Romanticism that has inflected culture since its very inception: can the aesthetic, and our critical engagement with the aesthetic, produce meaning that is, well, meaningful? The question begs too many qualifications, of course, not least of which is the often plaintive cry about the contingencies of predication; that is, meaning for whom, for what, and why do we even bother about it in the first place? We cannot predicate sure attributes of cultural purpose because, abstraction that it is,

we end in circular claims about the meaning of meaning. We are not quite circus animals chasing our respective tails, I hope, but this problem is consistently played out in the domain of pleasure, or at least of affective responsiveness. For surely we come to ask the question of cultural products only at the point in which we are radically invested in them: we profess in the domain of culture, and few professors in the humanities extricate their own modes of self-understanding from their professional preoccupations.

The issue, that is, defines us in banal ways too: after all the debates about the uses of pleasure, what can be said about our status as professional critics and scholars? (This is partly the issue that Thomas Pfau takes up polemically.) And must this question truly be allied with the more conceptually difficult one about the place of affective experience in aesthetic judgment? Both Pfau and Kelley are concerned to define the place of the aesthetic within a judgment that comprehends a relation between that which is meaningful for our interiority and that which is meaningful from the perspective of the socially iterable. Kelley finds reassurance in Hilary Putnam's recent re-thinking of philosophical realism, in which mind and world may be stitched together more thoroughly. But still more questions arise. Does the potential solipsism necessarily inherent in any aesthetic pleasure find a rapport, or a reciprocal production of meaning, with the empirical world? If Romanticism has a grasp upon the actual (to recall F.R. Leavis's famous indictment of Shelley) that is not merely weak, how do the actual and the pleasure of that aesthetic "grasp" signify to each other? These are the questions that I hope a brief consideration of Romanticism and philosophy in an historical age might open on to. The essays and counter-responses in this volume represent works in progress by Kelley and Pfau, and we invite our readers' input into their respective polemics. It was Foucault, of course, who re-ignited interest in the question, "What is Enlightenment," and the questions, "what is maturity?" and "what is modernity?" followed quick on its heels. But Foucault knew that the "aesthetics of existence" is interrogated precisely in the service of establishing “an ontology of ourselves", and the historicist passage between them must comprehend also the minutiae of expression. We need to know now what a mature reading in this post-enlightenment age of deeply vexed modernity can possibly mean. The access so vital to the final Foucault is an exercise of oneself; and if thought is an activity that yields a "game of truth" by which one undergoes change, then surely an interrogation of the technical "games" of poetics may be said to speak to a vital aspect of human need. Kelley's close analysis of John

CSAT Reading Comprehension

27 © Copyright 2011 Think & Learn Pvt. Ltd Copying or Reproduction without the prior permission of the author is prohibited

Clare's poetry is an instructive instance in this regard.

If poetic cadence, for example, resonates-or more to the point, if what we believe about the allure of cadence is that it answers to a rhythm essentially held within us-then we are, it is true, treading on structuralist ground: poetics touches us at the level of resonance sounding deep within us. But determining the historicity of formalist norms (this is just one instance of a possible avenue of exploration) is still fecund scholarly ground. What seems to have needlessly polarized the academy, however, is the assumption that poetic resonance must be interpreted as either ideological or, alternatively, structural in an essentialist, naively psychologised manner. But again, how could a psychological resonance not be, at least in some manner, participation within a dominant norm? Or at least, in what arenas were such assumptions ever challenged? The genealogy of the ideological ground of aesthetic compulsion still needs to take account of an aesthetic history. In this volume, Pfau and Kelley respond to one another partly in the terms of such issues (a response follows each essay). They help us find a way into a cultural context that does not, as it were, forgive the text merely its social determinations on the one hand, or fetishize its historical contingencies on the other. In some respects, what they articulate about Romanticism is nothing less than the uses (variously conceived) of its pleasures. 1) This passage could most probably have been sourced from a) A book written by Kelley and Pfau. b) A debate on romanticism by Kelley and Pfau. c) An article which talks about romanticism and culture, and views of different people like Pfau, Kelly on romanticism. d) A prelude to a composition which links romanticism and philosophy in a historical age. 2)According to the author, Kelley and Pfau wanted to establish which of the following things: A) Characterise the position of aesthetic within a judgement B) How do the actual and the pleasure of that aesthetic "Grasp" signify to each other C)The uses of the pleasures of Romanticism D)The place of Affective experience in aesthetic judgement a) A, B, C & D b) A, B & C c) A, B & D d) A & C 3)Why does the author use the statement “circus animals chasing our respective tails" a) The author wants to emphasise that this is a problem played out consistently in the domain of pleasure. b) The author wants to emphasise that we are facing a

problem with a circular claim on the meaning of meaning. c) The author wants to emphasise that this is a problem played out at least in the domain of affective responsiveness. d) Both a and c together. 4)Which of the following is probably not true with respect to the passage? a) Hilary Putnam supported Kelly’s Thought on aesthetics b) There were some authors who were criticising Shelley’s thoughts c) The author supports Kelly’s views of aesthetics position. d) None of the above 5)What is the main reason for Poetic cadence dividing the academy? a) Because Poetic cadence was interpreted as structural in an essentialist manner b) Because It answered to a rhythm essentially within us. c) Because it treaded on structuralist ground it was assumed to be structural in a naive manner d) None of the above 6) Who reignited the question -"What is maturity?” a) Kelly & Pfau in their book b) Foucalt c) John Clare d) F.R. Leavis

PASSAGE 2

For the agent, however, reason is the heart of the matter. And the heart of the matter is the reason for the civitas, whether it is vocalized or not. What is at stake in what I do is the kind of person I become. What is at stake in what we do is the kind of city we inhabit. In both the individual and the social variation of that mantra, familiar from virtue ethics, every action is the conclusion of a practical syllogism; it carries with it an argument, and the argument underwrites both character and civitas. The relationship is circular: character forms and is formed by every action, and each action tends to confirm the character of the agent. And the city forms and is formed by the characters it contains. When Paine says that the long habit of not thinking a thing wrong creates the superficial impression of its being right, he brings to our attention the fact that the long habit of not thinking a thing wrong makes it unlikely that we will think to change it. The shape of the city, like the shape of character, is a counterweight to change. On the positive side, this makes cities and characters relatively stable; and it gives us some idea what to expect of them if we have been paying attention. On the negative side, this

CSAT Reading Comprehension

28 © Copyright 2011 Think & Learn Pvt. Ltd Copying or Reproduction without the prior permission of the author is prohibited

renders characters and cities largely impervious to reason. Time makes more converts than reason, but time also tends, for better or worse, to confirm reasons of the heart that reason cannot know. What passes as stability may simply be inertia. Revolutionary theory turns on this question: is it stability or is it inertia? Either way, change is--and should be--difficult. For conservative theorists such as Burke, this translates into gradualism. The civitas changes in the manner of an organism, maturing in time and evolving across generations. Sudden change is the exception, not the rule. And, to a large extent, revolutionary theorists agree. Jefferson felt obliged to document a long pattern of abuse as justification for a single violent act. David Walker, writing with Jefferson in mind and partly in response to his Notes on the State of Virginia, followed the same pattern. Thoreau urged readers to let the ordinary friction of civil society pass and reserve disobedience for consistent affronts to human dignity. The African National Congress documented centuries of abuse before turning to armed resistance. Jefferson and Mao Zedong both asserted that every generation needed its own revolution, but, even so, they agreed that every revolution required reason. The whole world, for Jefferson, is a court before which the revolutionary has to make a case. That the New Left in the United States took up this Jeffersonian approach is reflected in its most simplified form by the chant "the whole world is watching" that framed demonstrations in Chicago in 1968. That the actions were (and are) called demonstrations suggests, at least, an audience and something to be demonstrated. Both revolution and war are put forward as rhetorical strategies within an argument that involves the world as a whole. Drawing the whole world into every act of violence may partly explain why "local" wars and revolutions have escalated into global conflicts. But my point here is to focus on rhetorical strategy in the context of an argument. Thoreau was convinced that no act was rhetorically insignificant, and both Gandhi and King followed him in this. The most revolutionary act in Gandhi's account was spinning the thread with which to make the clothes one wore. And this is critical to civil disobedience as a rhetorical strategy. One common thread in rational justifications for war and revolution is the documentation of violence and abuse against which war or revolution is a reaction. War and revolution are invariably depicted as last resorts: they are justified when there is nothing else to be done. This is hardly surprising, since the strongest argument

demonstrates a necessary conclusion. If the conclusion is necessary, then disagreement with it is nonsensical. In this regard, civil disobedience is a promising variation on a theme that includes the perpetual revolutions of Jefferson and Mao. Perpetual revolution suggests that no revolution is a conclusion: as a step in an argument, it can never be more than provisional. If resort to violence can be justified only by necessity, then it can never be justified. More to the point, it can never be more than provisionally justified. The rhetorical question (and, contrary to popular usage, there is no more important question to ask) is "what next?" Mao and Jefferson suspected that revolution led to revolution. As good revolutionaries, they went into this with their eyes open (at their best) and never let the revolutionary flame die down. In this regard, Tom Paine and Che Guevara were more consistent revolutionaries. But Thoreau was perhaps the most consistent revolutionary of all. Rather than fanning a revolutionary flame, he maintained that every act is part of a political argument. Compliance justifies the political context within which it takes place. Noncompliance undermines it. But while noncompliance undermines the political context within which it takes place, it implies another political context. Consciously and unconsciously, intentionally and unintentionally, it imagines and cultivates another political context. The constitution of the civitas is the ensemble of actions undertaken by those who inhabit it. 7)According to the author, what is the final outcome of a perpetual revolution included in civil disobedience? a) Perpetual revolution led to another revolution as it suggests that Revolution is not a conclusion. b) The visualization and Nurturing of a new political framework. c) Perpetual revolution led to another revolution as it suggests that Revolution can never be more than provisionally justified. d) Both a & c 8)Which of the following examples, if true, closely parallels one of the arguments brought about in the first paragraph of the passage? a) Sachin has never got out to a short ball. So he thinks he has the right technique to handle short balls and doesn’t want to change it. b) Dhoni has never missed a stumping while keeping. So he believes that his stance while keeping is correct. c) Murali has never been called for illegal action by umpires. So he doesn’t want to change his legal action now. d) Dickie bird has never given a wrong decision. So he sees no reason in changing his decision making process.

CSAT Reading Comprehension

29 © Copyright 2011 Think & Learn Pvt. Ltd Copying or Reproduction without the prior permission of the author is prohibited

9)With which of the following argument is the author non committal in the entire passage? a) Revolution and war are put forward as rhetorical strategies within an argument b) Sudden change is not a law but an exemption c) Perpetual revolution is not a conclusion but a step in the argument d) Local wars and revolutions escalating into global conflicts. 10)The following statements find support from the lines in the passage except a) David Walker, a revolutionary like Jefferson agreed with the views of Jefferson b) Thoreau, the most consistent revolutionary, urged people to reserve disobedience for consistent affronts to human dignity c) Mao and Jefferson were good revolutionaries who never let the revolutionary flame die down. d) Since war and revolution was deemed as a necessary conclusion to end violence and abuse disagreement with it was nonsensical.

PASSAGE 3

Writers’ invisibility has little or nothing to do with Fame, just as Fame has little or nothing to do with Literature. (Fame merits its capital F for its fickleness, Literature its capital L for its lastingness.) Thespians, celebrities and politicians, whose appetite for bottomless draughts of public acclaim, much of it manufactured, is beyond any normal measure, may feed hotly on Fame – but Fame is always a product of the present culture: topical and variable, hence ephemeral. Writers are made otherwise. What writers’ prize is simpler, quieter and more enduring than clamorous Fame: it is recognition. Fame, by and large, is an accountant’s category, tallied in Amazonian sales. Recognition, hushed and inherent in the silence of the page, is a reader’s category: its stealth is its wealth. And recognition itself can be fragile, a light too easily shuttered. Recall Henry James’s lamentation over his culminating New York Edition, with its considered revisions and invaluable prefaces: the mammoth work of a lifetime unheralded, unread, and unsold. That all this came to be munificently reversed is of no moment: the denizens of Parnassus are deaf to after-the-fact earthly notice; belatedness does them no good. Nothing is more poisonous to steady recognition than death: how often is a writer – lauded, fêted, bemedalled – plummeted into eclipse no more than a year or two after the final departure? Who nowadays speaks of

Bernard Malamud, once a diadem in the grand American trinity of Bellow-Roth-Malamud? Who thinks of Lionel Trilling, except with dismissive commemorative contempt? Already Norman Mailer is a distant unregretted noise and William Styron a mote in the middle distance (a phrase the nearly forgotten Max Beerbohm applied to the fading Henry James). As for poor befuddled mystical Jack Kerouac and declamatory fiddle-strumming mystical Allen Ginsberg, both are diminished to Documents of an Era: the stale turf of social historians and tedious professors of cultural studies. Yet these eruptions of sudden mufflings and posthumous silences must be ranked entirely apart from the forced muteness of living writers who work in minority languages, away from the klieg lights of the lingua franca, and whose oeuvres linger too often untranslated. The invisibility of recently dead writers is one thing, and can even, in certain cases (I would be pleased to name a few), bring relief; but the invisibility of the living is a different matter altogether, crucial to literary continuity. Political shunning – of writers who are made invisible, and also inaudible, by repressive design – results in what might be called public invisibility, rooted in external circumstance: the thuggish prejudices of gangsters who run rotted regimes, the vengeful prejudices of corrupt academics who propose intellectual boycotts, the shallow prejudices of the publishing lords of the currently dominant languages, and finally (reductio ad absurdum!) the ideologically narrow prejudices of some magazine editors. All these are rampant and scandalous and undermining of free expression. But what of an intrinsic, delicate and far more ubiquitous private invisibility? Vladimir Nabokov was once an invisible writer suffering from three of these unhappy conditions: the public, the private, and the linguistic. As an émigré fleeing the Bolshevik upheavals, and later as a refugee from the Nazis, he escaped the 20th century’s two great tyrannies. And as an émigré writing in Russian in Berlin and Paris, he remained invisible to nearly all but his exiled compatriots. Only on his arrival in America did the marginalising term “émigré” begin to vanish, replaced first by citizen and ultimately by American writer – since it was in America that the invisible became invincible. But Brian Boyd, in his intimate yet panoramic biography, recounts the difficulties, even in welcoming America, of invisible ink’s turning visible – not only in the protracted struggle for the publication of Lolita, but in the most liberal of literary journals. And here at last is the crux: writers are hidden beings. You have never met one – or, if you should ever believe you are seeing a writer, or having an argument with a writer, or listening to a talk by a writer, then you can be sure it is all a mistake. Inevitably, we are returned to Henry James, who

CSAT Reading Comprehension

30 © Copyright 2011 Think & Learn Pvt. Ltd Copying or Reproduction without the prior permission of the author is prohibited

long ago unriddled the conundrum of writers’ invisibility. In a story called “The Private Life”, Clare Vawdrey, a writer burdened by one of those peculiar Jamesian names (rhyming perhaps not accidentally with “tawdry”), is visible everywhere in every conceivable social situation. He is always available for a conversation or a stroll, always accessible, always pleasantly anecdotal, never remote or preoccupied. He has a light-minded bourgeois affability: “He talks, he circulates,” James’s narrator informs us, “he’s awfully popular, he flirts with you.” His work, as it happens, is the very opposite of his visible character: it is steeped in unalloyed greatness. One evening, while Vawdrey is loitering outdoors on a terrace, exchanging banalities with a companion, the narrator steals into Vawdrey’s room – only to discover him seated at his writing table in the dark, feverishly driving his pen. Since it is physically impossible for a material body to be in two places simultaneously, the narrator concludes that the social Vawdrey is a phantom, while the writer working in the dark is the real Vawdrey. “One is the genius,” he explains, “the other’s the bourgeois, and it’s only the bourgeois whom we personally know.” 11) Which of the following is best exemplified by the character Vawdrey in the passage? (a) Light-minded bourgeois affability. (b) Vawdrey is the answer to the writers’ invisibility. (c) The fact that a writer is the opposite of his perceptible character. (d) The premise that the writer is an apparition. (e) The truth that the writer is a brain. 12) According to the passage, the two tyrannies escaped by Nabokov were: (a) That he was an invisible writer and suffered linguistic problems. (b) That he fled from the Bolshevik revolution and the Nazi turmoil (c) That he was welcomed in America but also suffered a rejection. (d) That his invisibility extended to all and the fact that he wrote in Russian. (e) That as he began as an émigré and was replaced gradually as an American writer. 13) What, according to the author, is the reason for the invisibility of the living? a) It hampers literary continuity. b) The living are shy of the arc lights. c) Their work is not worthy of consideration. d) The language is difficult to follow. e) They are victims of parochialism.

14) The significance of admiration for writers is dissimilar from that of celebrities and politicians because (a) Fame is transient. (b) Fame offers immeasurable public approbation. (c) Writers look for deeper recognition. (d) Fame is clamorous. (e) Furtiveness is what the writers prefer

TEST-READING COMPREHENSION 08

No. of Questions-15 Time- 30 minutes

Passage 1

A meteor stream is composed of dust particles that have been ejected from a parent comet at a variety of velocities. These particles follow the same orbit as the parent comet, but due to their differing velocities they slowly gain on or fall behind the disintegrating comet until a shroud of dust surrounds the entire cometary orbit. Astronomers have hypothesized that a meteor stream should broaden with time as the dust particles’ individual orbits are perturbed by planetary gravitational fields. A recent computer-modeling experiment tested this hypothesis by tracking the influence of planetary gravitation over a projected 5,000-year period on the positions of a group of hypothetical dust particles. In the model, the particles were randomly distributed throughout a computer simulation of the orbit of an actual meteor stream, the Geminid. The researcher found, as expected, that the computer-model stream broadened with time. Conventional theories, however, predicted that the distribution of particles would be increasingly dense toward the center of a meteor stream. Surprisingly, the computer-model meteor stream gradually came to resemble a thick-walled, hollow pipe. Whenever the Earth passes through a meteor stream, a meteor shower occurs. Moving at a little over 1,500,000 miles per day around its orbit, the Earth would take, on average, just over a day to cross the hollow computer-model Geminid stream if the stream were 5,000 years old. Two brief periods of peak meteor activity during the shower would be observed, one as the Earth entered the thick-walled “pipe” and one as it exited. There is no reason why the Earth should always pass through the stream’s exact center, so the time interval between the two bursts of activity would vary from one year to the next. Has the predicted twin-peaked activity been observed for the actual yearly Geminid meteor shower? The Geminid data between 1970 and 1979 show just such a bifurcation, a secondary burst of meteor activity being clearly visible at an average of 19 horse (1,200,000 miles) after the first burst.

CSAT Reading Comprehension

31 © Copyright 2011 Think & Learn Pvt. Ltd Copying or Reproduction without the prior permission of the author is prohibited

The time intervals between the bursts suggest the actual Geminid stream is about 3,000 years old 1. The primary focus of the passage is on which of the following? (A) Comparing two scientific theories and contrasting the predictions that each would make concerning a natural phenomenon (B) Describing a new theoretical model and noting that it explains the nature of observations made of a particular natural phenomenon (C) Evaluating the results of a particular scientific experiment and suggesting further areas for research (D) Explaining how two different natural phenomena are related and demonstrating a way to measure them (E) Analyzing recent data derived from observations of an actual phenomenon and constructing a model to explain the data 2. According to the passage, which of the following is an accurate statement concerning meteor streams? (A) Meteor streams and comets start out with similar orbits, but only those of meteor streams are perturbed by planetary gravitation. (B) Meteor streams grow as dust particles are attracted by the gravitational fields of comets. (C) Meteor streams are composed of dust particles derived from comets. (D) Comets may be composed of several kinds of materials, while meteor streams consist only of large dust particles. (E) Once formed, meteor streams hasten the further disintegration of comets. 3. The author states that the research described in the first paragraph was undertaken in order to (A) determine the age of an actual meteor stream (B) Identify the various structural features of meteor streams (C) explore the nature of a particularly interesting meteor stream (D) test the hypothesis that meteor streams become broader as they age (E) show that a computer model could help in explaining actual astronomical data 4. It can be inferred from the passage that which of the following would most probably be observed during the Earth’s passage through a meteor stream if the conventional theories mentioned in line 18 were (A) Meteor activity would gradually increase to a single, intense peak, and then gradually decline.

(B) Meteor activity would be steady throughout the period of the meteor shower. (C) Meteor activity would rise to a peak at the beginning and at the end of the meteor shower. (D) Random bursts of very high meteor activity would be interspersed with periods of very little activity. (E) In years in which the Earth passed through only the outer areas of a meteor stream, meteor activity would be absent. 5. According to the passage, why do the dust particles in a meteor stream eventually surround a comet’s original orbit? (A) They are ejected by the comet at differing velocities. (B) Their orbits are uncontrolled by planetary gravitational fields. (C) They become part of the meteor stream at different times. (D) Their velocity slows over time. (E) Their ejection velocity is slower than that of the comet. 6. The passage suggests that which of the following is a prediction concerning meteor streams that can be derived from both the conventional theories mentioned in line 18 and the new computer-derived theory? (A) Dust particles in a meteor stream will usually be distributed evenly throughout any cross section of the steam. (B) The orbits of most meteor streams should cross the orbit of the Earth at some point and give rise to a meteor shower. (C) Over time the distribution of dust in a meteor stream will usually become denser at the outside edges of the stream than at the center. (D) Meteor showers caused by older by older meteor streams should be, on average, longer in duration than those caused by very young meteor streams. (E) The individual dust particles in older meteor streams should be, on average, smaller than those that compose younger meteor streams. 7. It can be inferred from the last paragraph of the passage that which of the following must be true of the Earth as it orbits the Sun? (A) Most meteor streams it encounters are more than 2,000 years old. (B) When passing through a meteor stream, it usually passes near to the stream’s center. (C) It crosses the Geminid meteor stream once every year. (D) It usually takes over a day to cross the actual Geminid meteor stream.

CSAT Reading Comprehension

32 © Copyright 2011 Think & Learn Pvt. Ltd Copying or Reproduction without the prior permission of the author is prohibited

(E) It accounts of most of the gravitational perturbation affecting the Geminid meteor stream. 8. Which of the following is an assumption underlying the last sentence of the passage? (A) In each of the years between 1970 and 1979, the Earth took exactly 19 hours to cross the Geminid meteor stream. (B) The comet associated with the Geminid meteor stream has totally disintegrated. (C) The Geminid meteor stream should continue to exist for at least 5,000 years. (D) The Geminid meteor stream has not broadened as rapidly as the conventional theories would have predicted. (E) The computer-model Geminid meteor stream provides an accurate representation of the development of the actual Geminid stream.

Passage 2

Most large corporations in the United States were once run by individual capitalists who owned enough stock to dominate the board of directors and dictate company policy. Because putting such large amounts of stock on the market would only depress its value, they could not sell out for a quick profit and instead had to concentrate on improving the long-term productivity of their companies. Today, with few exceptions, the stock of large United States corporations is held by large institutions-pension funds, for example-and because these institutions are prohibited by antitrust laws from owning a majority of a company's stock and from actively influencing a company's decision-making, they can enhance their wealth only by buying and selling stock in anticipation of fluctuations in its value. A minority shareholder is necessarily a short term trader. As a result, United States productivity is unlikely to improve unless shareholders and the managers of the companies in which they invest are encouraged to enhance long-term productivity (and hence long-term profitability), rather than simply to maximize short term profits. Since the return of the old-style capitalist is unlikely, today's short-term traders must be remade into tomorrow's long-term capitalistic investors. The legal limits that now prevent financial institutions from acquiring a dominant shareholding position in a corporation should be removed, and such institutions should be encouraged to take a more active role in the operations of the companies in which they invest. *In addition, any institution that holds twenty percent or more of a company's stock should be forced to give the public one day's notice of the intent to sell those shares. Unless the announced sale could be explained to the public on grounds other than anticipated future losses, the

value of the stock would plummet and, like the **old-time capitalists, major investors could cut their losses only by helping to restore their companies' productivity. Such measures would force financial institutions to become capitalists whose success depends not on trading shares at the propitious moment, but on increasing the productivity of the companies in which they invest. 9. In the passage, the author is primarily concerned with doing which of the following? (A) Comparing two different approaches to a problem (B) Describing a problem and proposing a solution (C) Defending an established method (D) Presenting data and drawing conclusions from the data (E) Comparing two different analyses of a current situation 10. It can be inferred from the passage that which of the following is true of majority shareholders in a corporation? (A) They make the corporation's operational management decisions. (B) They are not allowed to own more than fifty percent of the corporation's stock. (C) They cannot make quick profits by selling their stock in the corporation. (D) They are more interested in profits than in productivity. (E) They cannot sell any of their stock in the corporation without giving the public advance notice. 11. According to the passage, the purpose of the requirement suggested in lines * would be which of the following? (A) To encourage institutional stockholders to sell stock that they believe will decrease in value (B) To discourage institutional stockholders from intervening in the operation of a company whose stock they own (C) To discourage short-term profit-taking by institutional stockholders (D) To encourage a company's employees to take an active role in the ownership of stock in the company (E) To encourage investors to diversify their stock holdings 12. The author suggests that which of the following is a true statement about people who typify the “old style capitalist" referred to in line**? (A) They now rely on outdated management techniques. (B) They seldom engaged in short-term trading of the stock they owned. (C) They did not influence the investment policies of the corporations in which they invested. (D) They now play a much smaller role in the stock market as a result of antitrust legislation.

CSAT Reading Comprehension

33 © Copyright 2011 Think & Learn Pvt. Ltd Copying or Reproduction without the prior permission of the author is prohibited

(E) They were primarily concerned with maximizing the short-term profitability of the corporations in which they owned stock. 13. It can be inferred that the author makes which of the following assumptions about the businesses once controlled by individual capitalists? (A) These businesses were less profitable than are businesses today. (B) Improving long-term productivity led to increased profits. (C) Each business had only a few stockholders. (D) There was no short-term trading in the stock of these businesses. (E) Institutions owned no stock in these companies. 14. The author suggests that the role of large institutions as stockholders differs from that of the “old-style capitalist” in part because large institutions (A) invest in the stock of so many companies that they cannot focus attention on the affairs of any single corporation (B) are prohibited by law from owning a majority of a corporation’s stock (C) are influenced by brokers who advise against long-term ownership of stocks (D) are able to put large amounts of stock on the market without depressing the stock’s value (E) are attracted to the stocks of corporations that demonstrate long-term gains in productivity 15. The primary function of the second paragraph of the passage is to (A) identify problems (B) warn of consequence (C) explain effects (D) evaluate solutions (E) recommend actions

Passage 1

Traditionally, the first firm to commercialize a new technology has benefited from the unique opportunity to shape product definitions, forcing followers to adapt to a standard or invest in an unproven alternative. Today, however, the largest payoffs may go to companies that lead in developing integrated approaches for successful mass production and distribution. Producers of the Beta format for videocassette recorders (VCR’s), for example, were first to develop the VCR commercially in 1975, but producers of the rival VHS (Video Home System) format proved to be more successful at

forming strategic alliances with other producers and distributors to manufacture and market their VCR format Seeking to maintain exclusive control over VCR distribution. Beta producers were reluctant to form such alliances and eventually lost ground to VHS in the competition for the global VCR market. Despite Beta’s substantial technological head start and the fact that VHS was neither technically better nor cheaper than Beta, developers of VHS quickly turned a slight early lead in sales into a dominant position. Strategic alignments with producers of prerecorded tapes reinforced the VHS advantage. The perception among consumers that prerecorded tapes were more available in VHS format further expanded VHS’s share of the market. By the end of the 1980’s. Beta was no longer in production. 1. The passage is primarily concerned with which of the following? (A) Evaluating two competing technologies (B) Tracing the impact of a new technology by narrating a sequence of events (C) Reinterpreting an event from contemporary business history (D) illustrating a business strategy by means of a case history (E) Proposing an innovative approach to business planning 2. According to the passage, today’s successful firms, unlike successful firms in the past, may earn the greatest profits by (A) investing in research to produce cheaper versions of existing technology (B) being the first to market a competing technology (C) adapting rapidly to a technological standard previously set by a competing firm (D) establishing technological leadership in order to shape product definitions in advance of competing firms. (E) emphasizing the development of methods for the mass production and distribution of a new technology. 3. According to the passage, consumers began to develop a preference for VCR’s in the VHS format because they believed which of the following? (A) VCR’s in the VHS format were technically better than competing-format VCR’s. (B) VCR’s in the VHS format were less expensive than competing-format VCR’s. (C) VHS was the first standard format for VCR’s. (D) VHS prerecorded videotapes were more available than Beta-format tapes.

CSAT Reading Comprehension

34 © Copyright 2011 Think & Learn Pvt. Ltd Copying or Reproduction without the prior permission of the author is prohibited

(E) VCR’s in the Beta format would soon cease to be produced. 4. The author implies that one way that VHS producers won control over the VCR market was by (A) carefully restricting access to VCR technology B) giving up a slight early lead in VCR sales in order to improve long-term prospects. (C) retaining a strict monopoly on the production of prerecorded videotapes. (D) Sharing control of the marketing of VHS-format VCR’s (E) Sacrificing technological superiority over Beta format VCR’s in order to remain competitive in price. 5. The alignment of producers of VHS-format VCR’s with producers of prerecorded videotapes is most similar to which of the following? (A) The alignment of an automobile manufacturer with another automobile manufacturer to adopt a standard design for automobile engines. (B) The alignment of an automobile manufacturer with an automotive glass company whereby the manufacturer agrees to purchase automobile windshields only from that one glass company (C) The alignment of an automobile manufacturer with a petroleum company to ensure the widespread availability of the fuel required by a new type of engine developed by the manufacturer. (D) The alignment of an automobile manufacturer with its dealers to adopt a plan to improve automobile design. (E) The alignment of an automobile dealer with an automobile rental chain to adopt a strategy for an advertising campaign to promote a new type of automobile 6. Which of the following best describes the relation of the first paragraph to the passage as a whole? (A) It makes a general observation to be exemplified. (B) It outlines a process to be analyzed. (C) It poses a question to be answered. (D) It advances an argument to be disputed. (E) It introduces conflicting arguments to be reconciled.

TEST-READING COMPREHENSION 09

No. of Questions-10 Time- 20 minutes

Passage 1

Traditionally, the first firm to commercialize a new technology has benefited from the unique opportunity to shape product definitions, forcing followers to adapt to a standard or invest in an unproven alternative. Today,

however, the largest payoffs may go to companies that lead in developing integrated approaches for successful mass production and distribution. Producers of the Beta format for videocassette recorders (VCR’s), for example, were first to develop the VCR commercially in 1975, but producers of the rival VHS (Video Home System) format proved to be more successful at forming strategic alliances with other producers and distributors to manufacture and market their VCR format Seeking to maintain exclusive control over VCR distribution. Beta producers were reluctant to form such alliances and eventually lost ground to VHS in the competition for the global VCR market. Despite Beta’s substantial technological head start and the fact that VHS was neither technically better nor cheaper than Beta, developers of VHS quickly turned a slight early lead in sales into a dominant position. Strategic alignments with producers of prerecorded tapes reinforced the VHS advantage. The perception among consumers that prerecorded tapes were more available in VHS format further expanded VHS’s share of the market. By the end of the 1980’s. Beta was no longer in production. 1. The passage is primarily concerned with which of the following? (A) Evaluating two competing technologies (B) Tracing the impact of a new technology by narrating a sequence of events (C) Reinterpreting an event from contemporary business history (D) illustrating a business strategy by means of a case history (E) Proposing an innovative approach to business planning 2. According to the passage, today’s successful firms, unlike successful firms in the past, may earn the greatest profits by (A) investing in research to produce cheaper versions of existing technology (B) being the first to market a competing technology (C) adapting rapidly to a technological standard previously set by a competing firm (D) establishing technological leadership in order to shape product definitions in advance of competing firms. (E) emphasizing the development of methods for the mass production and distribution of a new technology. 3. According to the passage, consumers began to develop a preference for VCR’s in the VHS format because they believed which of the following?

CSAT Reading Comprehension

35 © Copyright 2011 Think & Learn Pvt. Ltd Copying or Reproduction without the prior permission of the author is prohibited

(A) VCR’s in the VHS format were technically better than competing-format VCR’s. (B) VCR’s in the VHS format were less expensive than competing-format VCR’s. (C) VHS was the first standard format for VCR’s. (D) VHS prerecorded videotapes were more available than Beta-format tapes. (E) VCR’s in the Beta format would soon cease to be produced. 4. The author implies that one way that VHS producers won control over the VCR market was by (A) carefully restricting access to VCR technology B) giving up a slight early lead in VCR sales in order to improve long-term prospects. (C) retaining a strict monopoly on the production of prerecorded videotapes. (D) Sharing control of the marketing of VHS-format VCR’s (E) Sacrificing technological superiority over Beta format VCR’s in order to remain competitive in price. 5. The alignment of producers of VHS-format VCR’s with producers of prerecorded videotapes is most similar to which of the following? (A) The alignment of an automobile manufacturer with another automobile manufacturer to adopt a standard design for automobile engines. (B) The alignment of an automobile manufacturer with an automotive glass company whereby the manufacturer agrees to purchase automobile windshields only from that one glass company (C) The alignment of an automobile manufacturer with a petroleum company to ensure the widespread availability of the fuel required by a new type of engine developed by the manufacturer. (D) The alignment of an automobile manufacturer with its dealers to adopt a plan to improve automobile design. (E) The alignment of an automobile dealer with an automobile rental chain to adopt a strategy for an advertising campaign to promote a new type of automobile 6. Which of the following best describes the relation of the first paragraph to the passage as a whole? (A) It makes a general observation to be exemplified. (B) It outlines a process to be analyzed. (C) It poses a question to be answered. (D) It advances an argument to be disputed. (E) It introduces conflicting arguments to be reconciled.

Passage 2

Shaw’s defense of a theater of ideas brought him up against both his great bugbears—commercialized art on the one hand and Art for Art’s Sake on the other. His teaching is that beauty is a by-product of other activity; that the artist writes out of moral passion (in forms varying from political conviction to religious zeal), not out of love of art; that the pursuit of art for its own sake is a form of self-indulgence as bad as any other sort of sensuality. In the end, the errors of “pure” art and of commercialized art are identical: they both appeal primarily to the senses. True art, on the other hand, is not merely a matter of pleasure. It may be unpleasant. A favorite Shavian metaphor for the function of the arts is that of tooth-pulling. Even if the patient is under laughing gas, the tooth is still pulled. The history of aesthetics affords more examples of a didactic than of a hedonist view. But Shaw’s didacticism takes an unusual turn in its application to the history of arts. If, as Shaw holds, ideas are a most important part of a work of art, and if, as he also holds, ideas go out of date, it follows that even the best works of art go out of date in some important respects and that the generally held view that great works are in all respects eternal is not shared by Shaw. In the preface to Three Plays for Puritans, he maintains that renewal in the arts means renewal in philosophy, that the first great artist who comes along after a renewal gives to the new philosophy full and final form, that subsequent artists, though even more gifted, can do nothing but refine upon the master without matching him. Shaw, whose essential modesty is as disarming as his pose of vanity is disconcerting, assigns to himself the role, not of the master, but of the pioneer, the role of a Marlowe rather than of a Shakespeare. “The whirligig of time will soon bring my audiences to my own point of view,” he writes, “and then the next Shakespeare that comes along will turn these petty tentatives of mine into masterpieces final for their epoch.” “Final for their epoch”—even Shakespearean masterpieces are not final beyond that. No one, says Shaw, will ever write a better tragedy than Lear or a better opera than Don Giovanni or a better music drama than Der Ring des Nibelungen; but just as essential to a play as this aesthetic merit is moral relevance which, if we take a naturalistic and historical view of morals, it loses, or partly loses, in time. Shaw, who has the courage of his historicism, consistently withstands the view that moral problems do not change, and argues therefore that for us modern literature and music form a Bible surpassing in significance the Hebrew Bible. That is Shaw’s anticipatory challenge to the neo-orthodoxy of today.

CSAT Reading Comprehension

36 © Copyright 2011 Think & Learn Pvt. Ltd Copying or Reproduction without the prior permission of the author is prohibited

7)According to the author, Shaw’s didacticism was unusual in that it was characterized by (A) idealism (B) historicism (C) hedonism (D) moralism 8)It can be inferred from the passage that Shaw would probably agree with all of the following statements about Shakespeare EXCEPT: (A) He wrote out of a moral passion. (B) He was the most profound and original thinker of his epoch. (C) He was a greater artist than Marlowe. (D) His Lear gives full and final form to the philosophy of his age. 9)Which of the following does the author cite as a contradiction in Shaw? (A) Whereas he pretended to be vain, he was actually modest. (B) Although he insisted that true art springs from moral passion, he rejected the notion that morals do not change. (C) He considered himself to be the pioneer of a new philosophy, but he hoped his audiences would eventually adopt his point of view. (D) On the one hand, he held that ideas are a most important part of a work of art; on the other hand, he believed that ideas go out of date. 10)The ideas attributed to Shaw in the passage suggest that he would most likely agree with which of the following statements? (A) A play cannot be comprehended fully without some knowledge and imaginative understanding of its context. (B) A great music drama like Der Ring des Nibelungen springs from a love of beauty, not from a love of art. (C) Morality is immutable; it is not something to be discussed and worked out. (D) Don Giovanni is a masterpiece because it is as relevant today as it was when it was created.

TEST-READING COMPREHENSION 10

No. of Questions-12 Time- 30 minutes

Passage 1

When A. Philip Randolph assumed the leadership of the Brotherhood of Sleeping Car Porters, he began a ten-year battle to win recognition from the Pullman Company, the largest private employer of Black people in the United States and the company that controlled the railroad

industry’s sleeping car and parlor service. (*)In 1935 the Brotherhood became the first Black union recognized by a major corporation. Randolph’s efforts in the battle helped transform the attitude of Black workers toward unions and toward themselves as an identifiable group; eventually, Randolph helped to weaken organized labor’s antagonism toward Black workers. In the Pullman contest Randolph faced formidable obstacles. (**)The first was Black workers’ understandable skepticism toward unions, which had historically barred Black workers from membership. An additional obstacle was the union that Pullman itself had formed, which weakened support among Black workers for an independent entity. The Brotherhood possessed a number of advantages, however, including Randolph’s own tactical abilities. In 1928 he took the bold step of threatening a strike against Pullman. Such a threat, on a national scale, under Black leadership, helped replace the stereotype of the Black worker as servant with the image of the Black worker as wage earner. In addition, the porters’ very isolation aided the Brotherhood. Porters were scattered throughout the country, sleeping in dormitories in Black communities; their segregated life protected the union’s internal communications from interception. That the porters were a homogeneous group working for a single employer with single labor policy, thus sharing the same grievances from city to city, also strengthened the Brotherhood and encouraged racial identity and solidarity as well. But it was only in the early 1930’s that federal legislation prohibiting a company from maintaining its own unions with company money eventually allowed the Brotherhood to become recognized as the porters’ representative. Not content with this triumph, Randolph brought the Brotherhood into the American Federation of Labor, where it became the equal of the Federation’s 105 other unions. He reasoned that as a member union, the Brotherhood would be in a better position to exert pressure on member unions that practiced race restrictions. Such restrictions were eventually found unconstitutional in 1944. 1. According to the passage, by 1935 the skepticism of Black workers toward unions was (A) unchanged except among Black employees of railroad-related industries. (B) reinforced by the actions of the Pullman Company’s union (C) mitigated by the efforts of Randolph

CSAT Reading Comprehension

37 © Copyright 2011 Think & Learn Pvt. Ltd Copying or Reproduction without the prior permission of the author is prohibited

(D) weakened by the opening up of many unions toBlack workers. (E) largely alleviated because of the policies of the American Federation of Labor. 2. In using the word “understandable” in the line indicated by: (**), the author most clearly conveys (A) sympathy with attempts by the Brotherhoodbetween 1925 and 1935 to establish an independent union. (B) concern that the obstacles faced by Randolph between 1925 and 1935 were indeed formidable (C) ambivalence about the significance of unions to most Black workers in the 1920’s. (D) appreciation of the attitude of many Black workers in the 1920’s toward unions. (E) regret at the historical attitude of unions towardBlack workers. 3. The passage suggests which of the following about the response of porters to the Pullman Company’s own union? (A) Few porters ever joined this union. (B) Some porters supported this union before 1935. (C) Porters, more than other Pullman employees, enthusiastically supported this union. (D) The porters’ response was most positive after 1935. (E) The porters’ response was unaffected by the general skepticism of Black workers concerning unions. 4. The passage suggests that if the grievances of porters in one part of the United States had been different from those of porters in another part of the country, which of the following would have been the case? (A) It would have been more difficult for the Pullman Company to have had a single labor policy. (B) It would have been more difficult for the Brotherhood to control its channels of communication. (C) It would have been more difficult for the Brotherhood to build its membership. (D) It would have been easier for the Pullman Company’s union to attract membership. (E) It would have been easier for the Brotherhood to threaten strikes. 5. The passage suggests that in the 1920’s a company in the United States was able to (A) use its own funds to set up a union (B) require its employees to join the company’s own union (C) develop a single labor policy for all its employees with little employee dissent.

(D) pressure its employees to contribute money to maintain the company’s own union (E) use its resources to prevent the passage of federal legislation that would have facilitated the formation of independent unions. 6. The passage supplies information concerning which of the following matters related to Randolph? (A) The steps he took to initiate the founding of the Brotherhood (B) His motivation for bringing the Brotherhood into the American Federation of Labor (C) The influence he had on the passage of legislation overturning race restrictions in 1944 (D) The influence he had on the passage of legislation to bar companies from financing their own unions (E) The success he and the Brotherhood had in influencing the policies of the other unions in the American Federation of Labor

Passage 2

Seeking a competitive advantage, some professional service firms(for example, firms providing advertising, accounting, or health care services) have considered offering unconditional guarantees of satisfaction. Such guarantees specify what clients can expect and what the firm will do if it fails to fulfill these expectations. Particularly with first-time clients, an unconditional guarantee can be an effective marketing tool if the client is very cautious, the firm’s fees are high, the negative consequences of bad service are grave, or business is difficult to obtain through referrals and word-of-mouth. However, an unconditional guarantee can sometimes hinder marketing efforts. (*)With its implication that failure is possible, the guarantee may, paradoxically, cause clients to doubt the service firm’s ability to deliver the promised level of service. It may conflict with a firm’s desire to appear sophisticated, or may even suggest that a firm is begging for business. In legal and health care services, it may mislead clients by suggesting that lawsuits or medical procedures will have guaranteed outcomes. (**)Indeed, professional service firms with outstanding reputations and performance to match have little to gain from offering unconditional guarantees. And any firm that implements an unconditional guarantee without undertaking a commensurate commitment to quality of service is merely employing a potentially costly marketing gimmick. 7. The primary function of the passage as a whole is to (A) account for the popularity of a practice

CSAT Reading Comprehension

38 © Copyright 2011 Think & Learn Pvt. Ltd Copying or Reproduction without the prior permission of the author is prohibited

(B) evaluate the utility of a practice (C) demonstrate how to institute a practice (D) weigh the ethics of using a strategy (E) explain the reasons for pursuing a strategy 8. All of the following are mentioned in the passage as circumstances in which professional service firms can benefit from offering an unconditional guarantee EXCEPT: (A) The firm is having difficulty retaining its clients of long standing. (B) The firm is having difficulty getting business through client recommendations. (C) The firm charges substantial fees for its services. (D) The adverse effects of poor performance by the firm are significant for the client. (E) The client is reluctant to incur risk. 9. Which of the following is cited in the passage as a goal of some professional service firms in offering unconditional guarantees of satisfaction? (A) A limit on the firm’s liability (B) Successful competition against other firms (C) Ability to justify fee increases (D) Attainment of an outstanding reputation in a field (E) Improvement in the quality of the firm’s service 10. The passage’s description of the issue raised by unconditional guarantees for health care or legal services most clearly implies that which of the following is true? (A) The legal and medical professions have standards of practice that would be violated by attempts to fulfill such unconditional guarantees. (B) The result of a lawsuit of medical procedure cannot necessarily be determined in advance by the professionals handling a client’s case. (C) The dignity of the legal and medical professions is undermined by any attempts at marketing of professional services, including unconditional guarantees. (D) Clients whose lawsuits or medical procedures have unsatisfactory outcomes cannot be adequately compensated by financial settlements alone. (E) Predicting the monetary cost of legal or health care services is more difficult than predicting the monetary cost of other types of professional services. 11. Which of the following hypothetical situations best exemplifies the potential problem noted in the second sentence of the second paragraph indicated by :(*)? (A) A physician’s unconditional guarantee of satisfaction encourages patients to sue for malpractice if they are unhappy with the treatment they receive.

(B) A lawyer’s unconditional guarantee of satisfaction makes clients suspect that the lawyer needs to find new clients quickly to increase the firm’s income. (C) A business consultant’s unconditional guarantee ofsatisfaction is undermined when the consultant failsto provide all of the services that are promised. (D) An architect’s unconditional guarantee of satisfaction makes clients wonder how often the architect’s buildings fail to please clients. (E) An accountant’s unconditional guarantee of satisfaction leads clients to believe that tax returns prepared by the accountant are certain to be accurate. 12. The passage most clearly implies which of the following about the professional service firms mentioned in line indicated by: (**) (A) They are unlikely to have offered unconditional guarantees of satisfaction in the past. (B) They are usually profitable enough to be able to compensate clients according to the terms of an unconditional guarantee. (C) They usually practice in fields in which the outcomes are predictable. (D) Their fees are usually more affordable than those charged by other professional service firms. (E) Their clients are usually already satisfied with the quality of service that is delivered.

CSAT Reading Comprehension

39 © Copyright 2011 Think & Learn Pvt. Ltd Copying or Reproduction without the prior permission of the author is prohibited

Solutions

TEST-RC 01

1. D is the best answer. This question requires you to identify the primary concern of the passage as a whole. The first paragraph presents a recent hypothesis about how caffeine affects behavior. The second paragraph describes an earlier and widely accepted hypothesis about how caffeine affects behavior, and then presents evidence that is not consistent with that hypothesis. The third and fourth paragraphs return to the newer hypothesis introduced in the first paragraph and provide “evidence and arguments” that support this alternative hypothesis. 2. D is the best answer. The passage states that adenosine “depresses neuron firing” by binding to specific receptors on neuronal membranes, which in turn inhibits the release of neurotransmitters. The passage describes Snyder et al’s hypothesis about caffeine. They propose that caffeine binds to specific receptors on neuronal membranes, which prevents adenosine from binding to those receptors and “allows the neurons to fire more readily than they otherwise would”. Therefore, according to Snyder et al, caffeine differs from adenosine in that caffeine permits neurotransmitter release when it is bound to adenosine receptors, whereas adenosine inhibits neruotransmitter release. 3. A is the best answer. The effects of IBMX are discussed in the last paragraph of the passage. IBMX apparently binds to adenosine-specific receptors on neuronal membranes, but, in contrast to the other caffeine derivatives that Snyder et al experimented with, IBMX depresses rather than stimulates mouse locomotion. Snyder et al respond to this experimental result by stating that IBMX has “mixed effects in the brain, a not unusual occurrence with psychoactive drugs”

4. E is the best answer. This question asks you to identify which compound, according to Snyder et al, does NOT bind to specific receptors in the brain. Phosphodiesterase, identified as an “enzyme that breaks down the chemical called cyclic AMP” is the only compound that is not identified as one that binds to specific receptors in the brain. 5. B is the best answer. This question asks you to identify information that is suggested rather than directly stated in the passage. To answer it, first look for the location in the passage of the information specified in the question. The A1 and A2 receptors are mentioned in the passage & it goes on to describe Snyder et al’s hypothesis about the effects of caffeine on behavior. They propose that caffeine, “which is structurally similar to adenosine,” is able to bind to A1 and A2 receptors in the brain, the same receptors that adenosine normally binds to. Thus, the passage suggests that the structural relationship between caffeine and adenosine may be partially responsible for caffeine’s ability to bind to A1 and A2 receptors. 6. B is the best answer. The first paragraph identifies two major problems faced by the archaeological profession: inadequate funding and illegal digging. The passage indicates that the author is going to suggest how to remedy both problems, thereby benefiting the archaeological profession. The author proceeds to propose allowing the sale of excavated artifacts and to explain how this would solve both problems . The author then supports the proposal by countering possible objections to it, and in the last paragraph explains how the proposal would curb illegal digging.Thus, the way information is organized in the passage indicates that the author’s purpose is to suggest that allowing the sale of excavated artifacts would provide funds for the archaeological profession and curb illegal digging. 7. E is the best answer. The question requires you to identify the answer choice that CANNOT be inferred from the passage. Nothing in the passage implies that duplicate artifacts exceed museum objects in quality. 8. E is the best answer. The disadvantages of storing artifacts in museum basements are discussed in the fifth paragraph. The passage states that “There is not enough money…to catalogue the finds” and declare that, as a result stored objects cannot be located. 9. A is the best answer. The author’s argument concerning the effect of the official sale of duplicate artifacts on illegal

CSAT Reading Comprehension

40 © Copyright 2011 Think & Learn Pvt. Ltd Copying or Reproduction without the prior permission of the author is prohibited

excavation appears in the passage where the author predicts that such official sale would reduce the demand for “the clandestine product.” The rhetorical question that follows indicates that the author finds it unlikely that any purchaser would prefer objects of unknown provenance to objects of known origin, or, to rephrase, the author assumes that most people would prefer to purchase objects of authenticated provenance, as this choice states. 10. C is the best answer. The author begins the third paragraph by saying “you might object…” in order to anticipate possible objections to the adoption of his proposal. In the next sentence the author asserts that “ancient artifacts…should be available for all to appreciate, not sold to the highest bidder”, acknowledging an opponent’s fear that individuals might be allowed to purchase objects that ought to be displayed in public institutions. This objection is paraphrased in this choice. 11. A is the best answer. The passage begins by indicating that the results of the SBA approach to aiding minority entrepreneurs “were disappointing” . The passage states that “MESBIC’s are now emerging as increasingly important financing sources for minority enterprises.” Much of the passage is devoted to supporting the author’s view that MESBIC’xhave the greater potential for success, and the last sentence in the passage confirms this view. 12. D is the best answer. In the second paragraph, the author describes the MESBIC approach as one in which “large corporations participate in the development of successful and stable minority businesses by making use of government-sponsored venture capital”. There is no indication in the passage that the SBA approach relies on the participation of large corporations. 13. C is the best answer. The author concludes that the results of the SBA approach “were disappointing”. Then he supports the conclusion by citing the fact that “Even 15 years after the program was implemented, minority business receipts were not quite two percent of the national economy’s total receipts.”. 14. C is the best answer. This question asks you to draw an inference about the SBA program. Although the passage does not actually state that the failure rate for SBA recipient businesses was higher than anticipated, in the first paragraph the author does state that the results of the SBA program were disappointing, in part because of the high failure rate among SBA-sponsored businesses. From this it

can be inferred that the anticipated failure rate was lower than the actual rate. 15. B is the best answer. The passage describes efforts undertaken in the 1960’s to and minority businesses and then describes MESBIC’s , a newer approach to such efforts.

TEST-RC 02

1. D is the best answer. The question requires you to recognize which of the choices is NOT mentioned in the passage as a way in which senior managers use intuition. The passage does not mention stipulating goals. 2. D is the best answer. The author asserts that the writers in question “display a poor grasp of what intuition is” . The next paragraph presents a view that, according to the author of the passage, characterizes intuition more accurately than the writers on management do. Isenberg’s research is specifically described as showing the ways in which managers use intuition. Therefore, what Isenberg correctly comprehends, and the writers in question misunderstand, is how managers use intuition, as this choice states. 3. D is the best answer. The question requires you to recognize which of the choices is NOT mentioned in the passage as a component of the classical model of decision analysis. Only this choice, “action undertaken in order to discover more information about a problem,” does not appear in the passage. 4. C is the best answer. The question requires you to compare behavior based on intuition with behavior based on formal decision analysis. This choice specifies that the manager who uses intuition incorporates action into the decision-making process, but the manager who uses formal analysis does not. This distinction is made in several places in the passage. The passage emphasizes that decision-making and action-taking are separate steps in formal decision analysis: “making a decision, and only then taking action.” On the other hand, those who use intuition “integrate action into the process of thinking”.Again, the author mentions that in the intuitive style of management, “ ‘thinking’ is inseparable from acting”, and “action is often part of defining the problem”. 5. E is the best answer. The question requires you to identify a statement that can be inferred from information in the passage but is not explicitly stated. The author asserts

CSAT Reading Comprehension

41 © Copyright 2011 Think & Learn Pvt. Ltd Copying or Reproduction without the prior permission of the author is prohibited

that intuitive managers can “move rapidly to engender a plausible solution” and that their intuition is based on “experience that builds skill”. This implies that the combination of skill and rapidity enables mangers to employ their practical experience more efficiently, as this choice states. 6. E is the best answer. The second and third paragraphs of the passage indicate that morphogenetic determinants are substances in the embryo that are activated after the egg has been fertilized and that “tell a cell what to become”. If, as the author asserts in the first paragraph, biologists have succeeded in dividing an embryo into two parts, each of which survives and develops into a normal embryo, it can be concluded that the quantity of morphogenetic determinants in the early embryo is greater than that required for the development of a single individual. 7. D is the best answer. In identifying the main topic of the passage, you must consider the passage as a whole. In the first paragraph, the author provides a historical context for the debate described in the second paragraph, concerning when and how the determination of embryo cells takes place. The third and forth paragraphs provide a specific example of the “Recent discoveries in molecular biology” that may lead to the resolution of that debate. 8. E is the best answer. According to the author, early investigators arrived at the conclusion that the cells of the embryo are undetermined because they “found that if they separated an invertebrate animal embryo into two parts at an early stage of its life, it would survive and develop as two normal embryos”. However, later biologists discovered that when an embryo was cut in places different from the one used by the early investigators, it did not form two whole embryos. Because the earlier biologists apparently arrived at their conclusion without attempting to cut an embryo in different planes, it would appear that they assumed, erroneously, that different ways of separating the embryos would not affect the fate of the two embryo parts. 9.A is the best answer. In the third paragraph, the author asserts that substances that function as morphogenetic determinants are located in the cytoplasm of the cell and become active after the cell is fertilized. In the fourth paragraph we learn that these substances are “maternal messenger RNA’s” and that they “direct, in large part, the synthesis of histones,” which, after being synthesized, “move into the cell nucleus”. Thus, it can be inferred that after the egg is fertilized, the initial production of histones occurs in the cytoplasm.

10.B is the best answer. Lines in the passage indicate that substances that function as morphogenetic determinants are inactive in the unfertilized egg and that when the egg is fertilized, they “become active and, presumably, govern the behavior of the genes they interact with.” In the fourth paragraph, we learn that these substances exert their control over the fate of the cell by directing “the synthesis of histones.” Because these histones cannot be synthesized until the substances that function as morphogenetic determinants become active, and because these substances do not become active until the egg is fertilized, it can be inferred that the synthesis of the histones is dependent on the fertilization of the egg. 11.C is the best answer. In his study of sea urchins, Gross “found that an unfertilized egg contains substances that function as morphogetic determinants.” The passage asserts that the “substances that Gross studied are maternal messenger RNA’s,” and we learn that these maternal messenger RNA’s can be found in “ a wide variety of organisms.” 12. D is the best answer. In the passage, the author states that ten percent of the Black workers in the South were employed in “manufacturing and mechanical pursuits” and then identifies “manufacturing and mechanical pursuits” as the general federal census category for industrial occupations in 1910. Thus, she indicates that she used the federal census as a source of information. 13. C is the best answer. To answer this question, you must first identify the author’s argument. The author argues that it is possible that Black migrants to the North were living and working in urban areas of the South rather in rural areas, as researchers had previously assumed. In the passage, the author states that it may be “surprising” that an employed population would relocate. Thus, the author anticipates an objection to her argument on the grounds that Black urban workers in the South would have been unlikely to leave an economically secure existence. She meets that objection by stating that “an explanation lies in the labor conditions then prevalent in the South”, and discusses the low wages that may have motivated Black workers to migrate north for higher pay. 14. A is the best answer. The author discusses wages in southern cities in the third paragraph. Lines state that an increase in the number of rural workers who migrated to southern cities after the collapse of the cotton industry led

CSAT Reading Comprehension

42 © Copyright 2011 Think & Learn Pvt. Ltd Copying or Reproduction without the prior permission of the author is prohibited

to increased competition for jobs and resulted in wages being pushed lower. 15. D is the best answer. This question asks you to identify the possible influences that motivated Black workers in their decision to migrate north, and then to recognize which of the choices is NOT mentioned as an influence on Black workers. This is the only option not mentioned in the passage as an influence that may have motivated southern Black workers to move north. 16. D is the best answer. The first paragraph describes a common assumption about the Great Migration, that the majority of migrants came from rural areas. It also restates the conclusion that is based on this assumption, that the subsequent economic difficulties of Black migrants in the North were a result of their unfamiliarity with urban life. In the second paragraph, the author states that the “question of who actually left the South” has never been adequately researched. She goes on to argue that Black migrants may actually have been from urban areas rather than rural areas, and thus that their subsequent economic problems in northern cities were not caused by their rural background. In making this argument, the author is challenging the “widely accepted explanation” presented in the first paragraph.

TEST-RC 03

1) The answer is (c) as the real reason behind the scientists’ annoyance is that they consider the Tsavo lions to be no more man-eaters than lions elsewhere, but the book has reinforced the view that they are more aggressive. 2) The answer is (d). The scientists realized that their earlier skepticism about the authenticity of the amateurs’ claims had been hasty as these claims had now been vindicated. 3) The answer is (c). Gnoske and Peterhanssays that Tsavo lions may be similar to the unmaned cave lions of the Pleistocence. Then the 5th line 3rd paragraph says that Tsavo lions have a reputation of ferocity. So logically, the unmaned cave lions of the Pleistocence are ferocious. But option (c) says that the unmaned cave lions of the Pleistocence are shown to be far less violent than believed therefore weakening the assumption. 4) The answer is (c). Other than (c) all of the rest will contribute to the image of the Tsavo being ferocious as they are stated in the passage. Option (a) and (c) are present in

the fourth paragraph of the passage. Option (d) is clearly stated in the fifth paragraph. 5) The answer is (a). The answer can be inferred from the last paragraph 2nd line onwards – "in the 1960s, the German auto companies developed plans to scrap the entire combustion engine…..why didn’t it go anywhere? Because auto executive understood pistons and carburettors, and would be loath to cannibalize their expertise, along with most of their factories." 6) The answer is (d). The answer can be inferred from the 6th paragraph – "fifty years after….largely cosmetic changes." 7) The answer is (b). The answer can be inferred from the last line of the 4th paragraph – “Henry Ford’s first car did 18 miles per hour and in 2001 the average speed in London was 17.5 miles per hour”. 8) The answer is (b). The answer can be inferred from the 4th and 6th paragraphs, where the author says that even after "a hundred years and tens of thousands of research hours later" little has changed in the auto industry and also changes in the aviation industry are incremental and largely cosmetic. 9) option (a) The passage is chiefly concerned with the existence of two kinds of dynamic psychology. 10) option (c) Dynamic psychology refers to understanding human disposition and behaviour related to it. 11) option (a) The term refers to the appearance of instinct and its subsequent transformation into habits. 12) Option (c) Only a connoisseur can appreciate better quality wines, taste is not written on labels 13) Option (a) Refer to the part some areas … have now produced a generation of growers using the varietal names on their labels. 14) Option (d) is the most substantiated reason to support Dr. Renaud’s findings.

CSAT Reading Comprehension

43 © Copyright 2011 Think & Learn Pvt. Ltd Copying or Reproduction without the prior permission of the author is prohibited

15)Option (b). Refer to the part “it is on every wine label … the name of the grape from which the wine is made … acquired a basic lexicon

TEST-RC 04

1.A is the best answer. In the first paragraph, the author cites an accidental-death case from nineteenth-century America in which the absence of economic contribution on the part of a deceased child was ruled sufficient grounds to deny the awarding of damages to the child’s parents. The author goes on to discuss how this case typified attitudes that persisted even into the twentieth century. It can be inferred from this that in nineteenth-century America, the chief consideration in determining damages in an accidental-death case was the deceased person’s earnings. There are no evidence in the passage to suggest that the factors in B, C, D and E were of primary concern in determining accidental-death damages in nineteenth-century America. 2. C is the best answer. In the second paragraph, the author describes how during the nineteenth century the concept of the “ ‘useful’ child who contributed to the family economy” gradually gave way to the present-day notion of the economically “useless” but emotionally “priceless” child. This new view of childhood was “well established among segments of the middle and upper classes by the mid-1800’s” and “spread throughout society in the late-nineteenth and early-twentieth centuries”. Thus in the early 1800’s, prior to the shift in the valuation of children, families valued the role children had to play in the family’s economic well-being. A and E describe attitude more in accord with the present-day view of childhood. B and D address issues that are not raised in the passage. 3. B is the best answer. According to the author, practitioners of the new “sociological economics” explain sociological phenomena “solely in terms of their economic determinants” and “tend to view all human behavior as directed primarily by the principle of maximizing economic gain’. This choice provides just such an economic explanation for the nineteenth-century rise in the cash value of children. A paraphrases Zelizer’s own explanation, which is at odds with that of the sociological economists. C uses social values and emotional factors to explain an even broader revaluation of individual worth. D uses an economic argument to explain the change, but here the economic factors at work are the result of a change. E provides a legal explanation for the change.

4. B is the best answer. In the first paragraph, the author contrasts two incidents that are said to exemplify the transformation in social values that forms the subject of Zelizer’s book. The second and third paragraphs consist of a brief history of that transformation, as Zelizer presents it, and an account of the factors she considers important in bringing it about. In the last paragraph, the author explains how Zelizer’s thesis differs from that of sociological economists. Thus, the passage serves primarily to present the central thesis of Zelizer’s book. A and E misrepresent the subject matter of the passage. D misrepresents the author’s approach. C is incorrect because although the passage does contrast two approaches, this contrast takes place only in the final paragraph. 5.C is the best answer. In the third paragraph, the author cites Zelizer’s contention that the new view of childhood that developed in nineteenth-century America was due in part to “the development of the companionate family (a family in which members were united by explicit bonds of love rather than duty)”. From this it can be inferred that the emotional bonds between family members became increasingly important during this period. There are no information in the passage to support the other answer choices. 6. D is the best answer. Although reform movements are mentioned ,the passage does not discuss attitudes or changes in attitudes toward those movements. This choice is therefore NOT among the influences Zelizer is said to regard as important in changing the assessment of children’s worth. A, B and C are mentioned as factors Zelizer regards as “critical in changing the assessment of children’s worth”. E is mentioned, which describe how the “sacralization” of children’s lives represented “a way of resisting what they <middle-class Americans> perceived as the relentless corruption of human values by the marketplace.” 7) Option (b) Refer to the part it is on every wine label … the name of the grape from which the wine is made … acquired a basic lexicon. 8) Option (d) is the most substantiated reason to support Dr.Renaud’s findings. 9) Option (a) Refer to the part some areas … have now produced a generation of growers using the varietal names on their labels.

CSAT Reading Comprehension

44 © Copyright 2011 Think & Learn Pvt. Ltd Copying or Reproduction without the prior permission of the author is prohibited

10) Option (b) The writer is using satire to mildly tease the French winemaker. 11) Option (c) Only a connoisseur can appreciate better quality wines, taste is not written on labels 12) A is the best answer. In the second paragraph, the author gives the percentag of workers who were unionized in different categories of the public sector in 1977. Forty-six percent of government professionals were unionized; this is greater than the percentage for any of the other categories of unionized workers from among the listed categories of public-sector workers. Therefore, professionals were more likely to belong to unions than were other public-sector workers. 13.E is the best answer.The author describes the reasoning behind the multioccupational unions’ attempt to achieve a fully unionized workplace. This reasoning is provided to explain why “the multioccupational unions would often try to organize them <clerical workers> regardless of the workers’ initial receptivity”. A helps to explain, but is not explained by, the attempt to achieve a fully unionized work force. An explanation for C is given in the first para. B and D are explained in the second and third paragraphs of the passage. 14.E is the best answer. The question asks what would strengthen the author’s claim that a new strategy for unionization has emerged since the mid-1970’s. The passage cites the appearance of the new strategy. The paragraphs that follow describe the changed circumstances that provided a context for such new strategies, explain precisely how these changed circumstances created a reason for new unionizing strategies. The author’s claim would be strengthened if it could be shown not only that there are such new circumstances, but that the old circumstances discussed in the first paragraph have become less important, further necessitating the adoption of a new strategy in place of an old strategy suitable to those older circumstances. 15. B is the best answer. In the first paragraph, the author describes the considerations relevant to a union’s attempt to organize a certain group of clerical workers prior to 1975.In the passage, the author notes the fact that most of these clerical workers were women, but does not suggest that this was an important consideration for unionizers.

16. C is the best answer. According to the author, “the women’s movement has succeeded in legitimizing the economic and political activism of women on their own behalf,” and this in turn has produced in women “a more positive attitude toward unions” Although other choices describe developments mentioned in the passage, none of these are said tohave been a consequence of the women’s movement. 17. D is the best answer. In the first paragraph of the passage, the author asserts that efforts to unionize public-sector clerical workers prior to 1975 were limited and then goes on to describe these limited efforts. In the second paragraph, the author asserts that a new strategy developed after 1975 and cites an increase in union membership among public-sector clerical workers. The author begins the last paragraph by asking what can explain this increase in union membership, and then proceeds to provide an explanation. Thus, the passage is primarily concerned with analyzing and explaining the increase in unionization among public-sector clerical workers.

Test-RC 05

1. The best answer is B. In the first sentence of the passage, the author characterizes the preoccupation of many United States companies with the search for legal protection from import competition as unfortunate. Then, the author explains that the “quest for import relief has hurt more companies than it has helped.” The third paragraph discusses one situation in which United States companies might experience such injury-when import relief laws are used against foreign subsidiaries of United States company-and the last paragraph provides a specific example of this situation. Thus, it can be inferred that the author’s primary concern is to warn about possible unintended negative consequences of applying trade laws. 2. The best answer is D. Bases for complaints to the International Trade Commission are discussed in the first paragraph. The author mentions the two specific kinds of complaints referred to in choices A and C (about imports benefiting from subsidies provided by foreign government sand about “dumping”), but goes on to conclude the paragraph with the comment that “the simple clam that an industry has been injured by imports is sufficient grounds to seek relief.” That a “simple claim” is “sufficient grounds to seek relief” suggests that the minimal basis for a complaint to the ITC is injury from the sale of imports in the United States, as stated in choice D. The situations in choices B and E are not discussed in the passage.

CSAT Reading Comprehension

45 © Copyright 2011 Think & Learn Pvt. Ltd Copying or Reproduction without the prior permission of the author is prohibited

3.The best answer is E. The last paragraph discusses a specific case in which a United States subsidiary of a Dutch conglomerate accused a Canadian branch of a United States company of “dumping” rock salt in the United States market. This incident is cited as “the most brazen case”of the problem stated in the previous paragraph: the use of import relief laws by foreign companies against of U.S. companies. No recommendations, discussion of unexpected results, or additional areas of research or concern are mentioned in the paragraph. Thus, choices A, B, C and D are not correct. 4.The best answer is D. The “danger” of import relief laws is stated : “that foreign companies will use import relief laws against the very companies the laws were designed to protect.” Import relief laws are the legal protection referred to in choice D. The passage does not mention the situations described in choice A, B, C, and E. 5.The best answer is D. In the passage, the author warns that it is “unlikely that a system of import relief laws will meet the strategic needs of all the units under the same parent company.” Thus, it can be inferred that the United States trade laws dealing with import relief will not necessarily help all units of a company, as stated in choice D. There is no indication in the passage that United States trade laws are expected to eliminate dumping, as is stated in choice A. Choice E is not discussed in the passage of the situations mentioned in choice B and C. 6.The best answer is B. The author states that “The bizarre aspect of the complaint was that a foreign conglomerate…was crying for help against a United States company…” It can be inferred that import relief laws were designed to protect United States companies from foreign competition. Thus, the lodging of a complaint by a foreign conglomerate against a United States company violated the intent of the laws. 7. The best answer is D. The first paragraph of the passage identifies a research method (recording life stories) and explains the method’s uses. The second and third paragraphs explain limitations of the method’s results. The final paragraph explains why the research method is useful despite its limitations. Choice A, B, and C are incorrect because only one research method is discussed, not two. Choice E can be eliminate because the passage does not discuss changing the method or adapting it to any other subject area.

8.The best answer is C. The Passage suggests that ethnologists “rarely spent enough time with the tribes they were observing.” Ethnologists who did not spend enough time with tribes they were observing were unlikely to be sufficiently familiar with the culture and customs of those tribes. Such ethnologists nevertheless attempted to describe the lives of tribal members. This attempt can be seen as analogous to the announcer’s attempt to describe the actions in a team sport with which he is unfamiliar. Choice A, B, and D can be eliminated because the passage does not suggest ethnologists deliberately withheld information. Choice E is incorrect because the passage does not mention any common ideas or positions held by both the ethnologists and the Native Americans. 9.The best answer is A, which paraphrases the passage’s assertion that life stores “are likely to throw more light on the working of the mind and emotions than any amount of speculation from an ethnologist or ethnological theorist from another culture” .Choice B is incorrect because the passage does not assess the difficulty of collecting life stories, and because the second paragraph discusses ways in which life stories became distorted. Choice C is incorrect because the passage does not specify how many research methods are available to ethnologists. Choice D can be eliminated because the third paragraph mentions distortion arising from ethnologists’ failure to recognize significant events in life stories. Choice E is incorrect because the second paragraph suggests that life stories would be more useful if collected by culturally knowledgeable investigators. 10.The best answer is E. In the third paragraph, the passage asserts that editors made their own decisions about which elements of the Native Americans’ life stories were important. It can therefore be inferred from the passage that reporting all of an informant’s information would help eliminate bias, because editing had involved subjective judgments about the intrinsic value of the information. Choice A, C, and D can be eliminated because the passage does not attribute bias to failures in adhering to ethnological theory, to translations into the researchers’ language, or problems in the numbers and content of question posed. Choice B is not supported because the second paragraph criticizes the emotion of the report, not that of the informant, for introducing bias. 11.The best answer is C. The passage describes a methodology, explain the methodology’s intended uses, criticizes the methodology’s accurateness and comprehensiveness, and reaffirms the methodology’s

CSAT Reading Comprehension

46 © Copyright 2011 Think & Learn Pvt. Ltd Copying or Reproduction without the prior permission of the author is prohibited

usefulness despite its limitations. Thus, the primary purpose of the passage is to evaluate or critique a methodology. 12. The best answer is B. The passage states that “Native Americans recognized that the essence of their lives could not be communicated in English,” that is, in the language of the ethnologists recording the life stories. Since this statement supports the idea that “much was inevitably lost,” it can be inferred that the informants used a language other than that used to record their life stories. Choice A is incorrect because, in the second paragraph, the investigators are criticized for lacking familiarity with the cultures they studies. Choice C is incorrect because ethnologists recorded life stories to “supplement their own field observations”. Choice D is incorrect because the passage indicates that life stories were edited; choice E is incorrect because the passage provides no information about guidelines used by the researchers. 13) Option (c) The lines “And indeed… I presume?” spell out the circle of concern which is limited to the immediate and the temporal world, nowhere do the lines depict any issues with the larger picture in life. The author is in a ‘status quo’ and deciding what to do. 14) Option (d) The passage stresses on the fact that “there will be time” for everything. 15) Option (d) The phrase “muttering retreats” makes option (d) correct. 16) Option (b) In the olden days, ether was used to anesthetize patients.

TEST-RC 06

1.The best answer is B. The passage states each of the five well-know plant hormones ‘has more than one effect on the growth and development of plants” and that, for this reason, “they are not very useful in artificially controlling the growth of crops” .Choice A is not correct because the passage describes some of the functions performed by the hormone auxin. Choice E is consistent with information presented in the passage, but by emphasizing the specific effect hormone have at the cellular level rather than the multiplicity of effects they have on the entire plant, E fails to prove the reason stated in the passage that the five

hormones are not useful in controlling the growth of crops. Neither C nor D is suggested by anything in the passage. 2.The best answer is D. According to the passage, “The pleiotropy of the five well-studies plant hormones is somewhat analogous to that of certain hormones in animals” .The example given involves certain hypothalamic hormones that “stimulate the anterior lobe of the pituitary gland to synthesize and release many different hormones, one of which stimulates the release of hormones from the adrenal cortex”. These hormones in turn “have specific effects on target organs all over the body”. This “hierarchy of hormones,” as the author calls it, “may also exist in plants” (line 35), where the five pleiotropic hormones may “function by activating the enzymes that release … more specific chemical messengers”. Thus, hypothalamic hormones in animals and the five major hormones in plants occupy a similar place in the respective organisms” hormonal hierarchy. 3. The best answer is A. The last paragraph characterizes oligosaccharins as “specific chemical messengers”. The passage indicates that these chemical messengers are “specific” in that, unlike the pleiotropic hormones, they are likely to have particular effects on particular plant cells. Choice A is correct because it is the only answer choice that describes an effect on a specific aspect of plant growth and development: stimulating a particular plant cell to become part of a plant’s root system. Choice B and C are incorrect because the last paragraph indicates that enzymes activate the release of oligosaccharins. Choice D is incorrect because, although oligosaccharins do affect the activity of the gene complement of a particular cell, they do not duplicate that complement. Choice E is incorrect because the second paragraph indicates that an oligosaccharin has a specific effect rather than multiple effects on plant cells. 4. The best answer is E. The second paragraph states that the five major plant hormones, including auxin, are pleiotropic and indicates that each pleiotropic hormone has “more than one effect on the growth and development of plants”. The effects of auxin are then listed in detail to provide an example of the different effects a pleiotropic hormone can have on a plant. Thus, the specific effects of auxin are mentioned to illustrate the concept of pleiotropy as it is exhibited by plant hormones. Choice C can be eliminated because the specific effects that auxin has on plant development are not discussed in the context of the hierarchy of hormones. Choice A, B, and D are incorrect because they cite topics that are not discussed in the passage.

CSAT Reading Comprehension

47 © Copyright 2011 Think & Learn Pvt. Ltd Copying or Reproduction without the prior permission of the author is prohibited

5.The best answer is E. The first paragraph states that plant cells “differentiate and form structures” when a “complex system of chemical messengers” activates a “small subset of the genes in a particular kind of cell” .In the passage, the author elaborates on the hormonal system in plants by indicating that the pleiotropic plant hormones activate enzymes, which in turn release oligosaccharins-the “more specific chemical messengers”. The second paragraph indicated these specific chemical messengers have specific effects on plant development. Thus, the passage indicates that it is the oligosaccharins that directly influence the development of a plant cell by controlling the expression of a plant cell’s genes. Choices C and D are incorrect because the oligosaccharins are themselves specific chemical messengers and are not said to produce any hormones. The passage provides no information to support A or B. 6.The best answer is C. The passage states that because each pleiotropic hormone has so many different effects on a plant, pleiotropic hormones “are not very useful in artificially controlling the growth of crops”. In contrast, the passage indicates that oligosaccharins have specific effects on the growth and development of plants. Thus, in comparison to the pleiotropic hormones, oligosaccharins could potentially be effective in artificially controlling specific aspects of crop development. Choices A, B, D, and E can be eliminated because they describe functions that are not attributed in the passage either to the pleiotropic hormones or to oligosaccharins. 7. This question asks you to identify the primary concern of the passage. The best answer is C. According to the first paragraph, the passage is about two different assessments of Florence Nightingale’s career. The first paragraph summarizes one of these assessments; the second paragraph presents a contrasting account of Nightingale’s career that contradicts the central point of the first account. Choice A is incorrect. Although the passage discusses various aspects of Florence Nightingale’s involvement in the field of nursing, it does not mention any innovations that she introduced to that field. Choice B is not correct because the passage does not discuss approaches to the writing of historical biography. Choice D is also incorrect. Although the passage refers to the specific problems of military medicine during the Crimean War and to the poor living conditions of British soldiers after the war, the passage does not discuss the broader, more general issue of the quality of health care in nineteenth-century England. And choice E is not the answer because the passage does not mention the effects of the Crimean War on developments in health care.

8. This question asks you to identify a contribution that the editors of Nightingale’s letters attribute to her. The best answer is E. In the second paragraph, several of Nightingale’s post-war accomplishments that are highlighted by the editors of her letters are mentioned. In the passage, her contribution to the creation of an organization for monitoring the peacetime living conditions of British soldiers is mentioned as one of these. Choice A is not correct. The editors of Nightingale’s letters cite the relatively high death rate of British soldiers after the Crimean War, but they do not mention their survival rate during the war. Choice B is incorrect, because the passage does not provide any information about the curriculum of the nurses’ training hospital that Nightingale founded. Choice C is also not the correct answer. The passage does not mention women doctors, only women nurses. And choice D is incorrect because there is no indication in the passage that the nurse’s training hospital that Nightingale founded was at a university or that it was the first of its kind. 9. To answer this question, you must use information contained in the passage to infer something about Nightingale’s relationship with the British public of her day. The best answer is A. Lines refer to Nightingale’s “heroic reputation”; & to “nightingale’s place in the national pantheon”; and the para discusses her persuasiveness with the British government and her fund-raising successes. From this information it can be inferred that Nightingale was highly respected, as evidenced by both popular and governmental support for her projects. Choices B, C, and D are incorrect for the same reason: each one refers to an element of social or governmental opposition or resistance to Nightingale’s ideas, none of which is mentioned or suggested by the passage. Choice E is also incorrect. The information in the passage contradicts the notion that Nightingale was “quickly forgotten.” To the contrary, the passage discusses the “famous British nurse Florence Nightingale”, her “heroic reputation”, and her “place in the national pantheon” , as well as her “eminent place among the ranks of social pioneers” . 10. The question asks you to draw a conclusion about sanitary conditions in Britain after the Crimean War that is suggested, rather than stated expressly, in the passage. The best answer is D. In the second paragraph Nightingale’s efforts to reform sanitary conditions in Britain are illustrated by her response to the death rate among enlisted men in British barracks, which is described as unusually high relative to that of neighboring civilian populations. From

CSAT Reading Comprehension

48 © Copyright 2011 Think & Learn Pvt. Ltd Copying or Reproduction without the prior permission of the author is prohibited

this it can be inferred that sanitary conditions in the barracks were worse than in these civilian populations. Choices A, B and C are incorrect: in each, a comparison is made between sanitary conditions in post-war Britain and sanitary conditions elsewhere or at other times. However, because the passage provides no basis on which to make any of these comparisons, all three of these choices are incorrect. The passage does not mention sanitary conditions “in other parts of the world,” as in Choice A; “before the war,” as in choice B; or in “rural areas” as compared with “urban centers”, as in choice C. Choice E is also incorrect, because the passage provides no information about the general state of sanitary conditions “throughout England”. 11. This question asks you to select a statement about the two contrasting accounts of Nightingale’s importance with which the author of the passage would be most likely to agree. The best answer is C. In the last paragraph, the author concedes that “Nightingale may not have achieved all of her goals during the Crimean War.” This is consistent with Summers’ view that Nightingale’s importance during the war has been exaggerated ,but the author of the passage nonetheless describes Nightingale as a great social pioneer because of her vision and achievements. These achievements, which the second paragraph states occurred primarily after the Crimean War, apparently did not influence Summers’ interpretation of Nightingale’s importance. Given the author’s favorable assessment of Nightingale’s reputation, it is likely that the author would agree that Summers’ interpretation ignores this important evidence. Choice A is not the correct answer. Although, in the passage, the author concedes that Summers may be correct in her assessment of Nightingale’s wartime achievements, nothing is said in the passage about Summers’ discussion, if any, of Nightingale’s postwar influence or activities. Choice B is also incorrect. The passage cites the editors’ collection of Nightingale’s letters as evidence of Nightingale’s “brilliance and creativity.” In addition, in light of the author’s statement that Nightingale has earned “an eminent place among the ranks of social pioneers”,there is no reason to think the author would agree that the editors exaggerated Nightingale’s influence on later generations. Choice D is not the correct answer. In the last paragraph of the passage, the author refers to Nightingale’s letters as evidence of her “brilliance and creativity” ,and as the basis for a conclusion that Nightingale has earned “an eminent place among the ranks of social pioneers” .It is therefore highly unlikely that the author believes that the editors of these letters have “mistakenly propagated” outdated notions or impeded a balanced assessment of Nightingale’s role. Choice E is also

incorrect. In the last paragraph of the passage, the author states that “the evidence of Nightingale’s letters supports continued respect for Nightingale’ brilliance and creativity.” Summers, on the other hand, seeks in her book to “debunk” Nightingale’s “heroic reputation”. Rather than supporting Summers’ conclusions about Nightingale, the evidence of Nightingale’s letters contradicts them. 12. The best answer is A. In the last paragraph of the passage, the author presents two examples of Nightingale’s “brilliance and creativity.” In the first of these, the author compares Nightingale to “a modern educator” for counseling a village schoolmaster to encourage children’s powers of observation. The fact that the author believes that this is evidence of Nightingale’s creativity suggests that it was unusual at that time to emphasize developing children’s ability to observe. Choice B is not the correct answer. Nightingale’s efforts to improve conditions in British military barracks are not cited as evidence of her creativity, nor is it suggested that Nightingale’s counseling a village schoolmaster, not enlisting schoolmasters’ help ;moreover, nothing in the passage suggests that educators had failed to enlist such help prior to the incident the author describes. Choice D is incorrect: although the author cites Nightingale’s contributions to the care of the needy , the passage does not suggest that no organized help for the needy existed before Nightingale began her work. And choice E is incorrect, because although Nightingale’s cost-accounting system is presented in the passage as having made a lasting contribution to the British Army’s medical services, the passage never suggests that before Nightingale the Army lacked a cost-accounting system. 13. This question asks you to identify the author’s primary concern in the last paragraph of the passage. The best answer is D. In the last paragraph, the author cites examples of Nightingale’s achievements to support the author’s conclusion that Nightingale’s achievements to support the Author’s conclusion that Nightingale is worthy of respect and has earned “an eminent place among the ranks of social pioneers” Choice A is incorrect. The third paragraph does not summarize the arguments presented in the first two paragraphs. Choice B is also not the correct answer: in the third paragraph, the author expresses essential agreement with the positive view of Nightingale’s career described in the second paragraph. Choice C is incorrect because in the last paragraph the author does not analyze the weakness of any evidence cited elsewhere in the passage. And choice E is not the correct answer because the author does not correct any factual errors in the two works under review.

CSAT Reading Comprehension

49 © Copyright 2011 Think & Learn Pvt. Ltd Copying or Reproduction without the prior permission of the author is prohibited

Test-RC 07

1) Option (d) The key to this question lies in the last 3 lines of the first paragraph "These are the questions that I hope a brief consideration of Romanticism and philosophy in an historical age might open on to. The essays and counter-responses in this volume represent works in progress by Kelly and Pfau, and we invite our readers' input into their respective polemics." 2) Option (d) The question asks about the opinions of Kelly and Pfau. In the first paragraph look at the line "Both Pfau and Kelley are concerned to define the place of the aesthetic within a judgment " .Also the last line of last paragraph states that "In some respects, what they articulate about Romanticism is nothing less than the uses (variously conceived) of its pleasures" These statements are brought in A) and C). Both B) and C) are the views of the author 3) Option (b) Please look at these lines in the first paragraph “we end in circular claims about the meaning of meaning. We are not quite circus animals chasing our respective tails" .This clearly gives the answer to the question. All the other 3 options are junk answers. 4) Option (c) Option (a) is wrong because of this line in the passage "Kelly finds reassurance in Hilary Putnam's recent re-thinking of philosophical realism" Option (b) is wrong because of this line in the passage "This is partly the issue that Thomas Pfau takes up polemically." This indicates that some people/ authors were against Pfau's thoughts. Option (c) does not find support in the passage. Look at the line "But still more questions arise. Does the potential solipsism necessarily inherent in any aesthetic pleasure find a rapport, or a reciprocal production of meaning, with the empirical world? If Romanticism has a grasp upon the actual that is not merely weak, how do the actual and the pleasure of that aesthetic "grasp" signify to each other? " It looks as if the author doesn’t fully agree with Kelly's thoughts.

5) Option (c) gives the perfect answer. The first reason is that poetic cadence treaded on structuralist ground which made many academicians to believe that it was structural. Look at these lines "it is true, treading on structuralist ground:" and " is the assumption that poetic resonance must be interpreted as either ideological or, alternatively, structural in an essentialist, naively psychologised manner" All the other answers are not complete. 6) option (b). Direct question from the first para. 7)Option (b) Look at these lines from the last paragraph "civil disobedience is a promising variation on a theme that includes the perpetual revolutions " and "Consciously and unconsciously, intentionally and unintentionally, it imagines and cultivates another political context" The question is what is the final outcome of a perpetual revolution.It finally creates a new political context . So answer (b) is correct. All the other answer options are the intermediate conclusions and not the final outcome. 8]Option (c)This question is with reference with the line "When Paine says that the long habit of not thinking a thing wrong creates the superficial impression of its being right, he brings to our attention the fact that the long habit of not thinking a thing wrong makes it unlikely that we will think to change it." So there are 2 consequences of not thinking a thing wrong 1) You think you are right 2) You are unlikely to change Murali thinks his action is legal and he doesn’t want to change .Only option c) illustrates both the consequences. 9]Option (d) Look at this line in the passage "Drawing the whole world into every act of violence may partly explain why "local" wars and revolutions have escalated into global conflicts. But my point here is to focus on rhetorical strategy in the context of an argument." The author himself doesn’t want to comment on option (D) .Every other option is discussed in the passage by the author with examples. 10]Option (a) The author doesn’t state anywhere that David walker is a revolutionary. The only thing the author states is "David Walker, writing with Jefferson in mind and partly in response to his Notes on the State of Virginia, followed the same pattern"

CSAT Reading Comprehension

50 © Copyright 2011 Think & Learn Pvt. Ltd Copying or Reproduction without the prior permission of the author is prohibited

11) Option (c) The last paragraph that talks about Vawdrey emphasizes the fact that the writer’s work is steeped in greatness and people get to know of only the outer layer, the real personality of the writers comes through only in their work. This is represented only in option (c). Option (a) talks only about one side of the writer’s character and hence cannot be the answer. Option (b) also talks about only one side of the character. Option (d) does not make any sense. Option (c) is the correct answer 12) Option (b) The passage mentions that Nabokov escaped the 20th century’s greatest tyrannies which were the Bolshevik upheavals and the Nazi persecution. 13) Option (e) Refer to para 3 where the author feels that the invisibility stems from varied prejudices. 14) Option (c) Option (c) can be easily inferred from para 1.

TEST –RC 08

1.This question asks you to identify the primary focus of the passage. The best answer is B. The author describes the new theoretical model in the first paragraph; in the final paragraph the author states that the data obtained from actual observations, which are discussed in the second and third paragraphs, is consistent with the new theoretical model. Choice A is not correct; the computer model confirmed the astronomers’ hypothesis that meteor streams broaden with time, and although the model yielded an unexpected result, the passage does not contrast the predictions yielded by competing theories. Choice C and D are not correct because the passage makes no reference to further areas for research, and only a single phenomenon is described in the passage. And choice E is not correct because it reverses the order of events. The model yielded a prediction that was subsequently confirmed by observational data, the model was not constructed to explain the data. 2.This question asks you to identify an accurate statement about meteor streams. Choice C, the best answer, restates information about the composition of meteor streams from the first sentence of the passage. Choice A is not correct. The passage discusses the influence of planetary gravitation on meteor streams but says nothing about its influence on the orbits of comets. According to the passage, it is

planetary gravitation, not the gravitational fields of comets, that causes meteor streams to increase in size, so choice B is not correct. And choice D and E are not correct answers because the passage says nothing about the composition of comets or the role that meteor streams play in their further disintegration. 3. This question asks what the author says about the purpose of the research described in the first paragraph. The best answer is D. According to the author, the purpose of the computer-modeling experiment was to test the hypothesis that meteor streams broaden with time. Choice A is not correct; although the observational data described in the last paragraph allowed scientists to estimate the age of the Geminid stream, this data was analyzed to confirm a surprising prediction made by the computer model. This analysis was not part of the original experiment. Choice B is also incorrect. Although the experiment yielded a surprising prediction about a particular feature of meteor streams, the purpose of the experiment was to determine whether meteor streams broaden with time, not to identify the various structural features of meteor streams. Choice C is not correct because the experiment was undertaken to test a general hypothesis about meteor streams. It was not undertaken to explore the nature of any particular meteor stream, and the passage never suggests that the actual meteor streams used in the computer model was “particularly interesting.” Choice E is not correct. Although the computer model did confirm the astronomers’ hypothesis, the purpose of the experiment was not to show that such models are useful. 4.This question asks you to make an inference about what would most probably be observed during the Earth’s passage through a meteor stream if the conventional theories mentioned in the passage were correct. According to line 18-20, the conventional theories predicted that the meteor stream would be most dense at the center. The computer model, one the other hand, predicted that a meteor stream would come to resemble a thick-walled, hollow pipe.The passage states that, if the computer model were correct, two peak periods of meteor activity would be observed as the Earth passed through the walls of the “pipe” . According to the passage, observational data confirmed the prediction of the computer model. If, on the other hand, the conventional theories were correct, it can be inferred that a bifurcation of meteor activity would not be observed; instead, it can be inferred that scientists would expect to observe a single peak of meteor activity as the Earth passed through the dense center of the stream. Choice A identifies this single peak of activity as the most

CSAT Reading Comprehension

51 © Copyright 2011 Think & Learn Pvt. Ltd Copying or Reproduction without the prior permission of the author is prohibited

likely observation if the conventional theories were correct. Choice B and D are not correct because they describe meteor activity that is either steady or erratic, neither of which is consistent with the conventional theories. Choice C describes meteor activity more in line with the bifurcation predicted by the computer model, rather than the single peak of activity that the conventional theories would suggest. Choice E is incorrect because the passage says that meteor showers occur whenever the Earth passes through a meteor stream; it cannot be inferred that either theory would predict otherwise. 5.This question asks for the reason given in the passage for a characteristic feature of meteor streams. According to the passage, the dust particles in a meteor stream eventually surround a comet’s original orbit because of the different velocities at which they are ejected, as stated in choice A, the best answer. Choice B is directly contradicted by information in the passage .The other answer choices re incorrect because the passage does not say that the dust particles become part of the meteor stream at different times, or that their velocity slows over time, or that their ejection velocity is slower than that of the comet. 6. This question asks you to identify a prediction that can be derived from both the conventional theories about meteor streams and the new computer-derived model. You must base your answer on information that is suggested by, but not expressly stated in, the passage. According to the passage, the conventional theories hypothesized that meteor streams should broaden with time, and the computer simulation confirmed this hypothesis. The passage also suggests that the time it takes for the Earth to cross a meteor stream (and, by implication, the duration of the resulting meteor shower) is directly related to the breadth of the stream. From these pieces of information, which are supported by both the conventional theories and the new computer-derived theory, it can be inferred that on average the meteor showers caused by older (and therefore broader) meteor streams would be longer in duration than those caused by very young (and therefore narrower) meteor streams, as stated in D, the best answer. Choice A is incorrect because it contradicts the predictions of both the conventional theories (that the particles will be most dense at the center of the stream) and the computer model (that the stream will resemble a thick-walled, hollow pipe). Choice C is also incorrect because it is inconsistent with the conventional theories that suggested the distribution of dust in a meteor stream is denser at the center. And choices B and E are incorrect because the theories discussed in the passage do not suggest anything about the likelihood that

the Earth’s orbit will cross that of any particular meteor stream, nor do they suggest anything about the size of the dust particles that compose meteor streams. 7.This question asks you to draw an inference from information in the last paragraph of the passage. The best answer is C. According to the passage, the Geminid meteor shower occurs yearly; because meteor showers occur whenever the Earth passes through a meteor stream, one can infer that the Earth crosses the Geminid stream once every year. Choice A is incorrect because the passage provides no information from which to generalize about the age of meteor streams. Choice B, which is directly contradicted by the passage, is also incorrect. Choice D is incorrect. The passage says that the Earth would take just over a day to cross the stream if the stream were 5,000 years old. However, in the next few lines the passage states that in fact an average of only 19 houses elapsed between the time that the Earth entered the stream until the time that it exited, leading researchers to conclude that the stream is only about 3,000 years old. Choice E is incorrect because the passage says only that planetary gravitational fields perturb the orbits of dust particles in a meteor stream; it does not say that the effect of the Earth’s gravitation is greater than that of other planets. 8.This question asks you to identify an assumption underlying the last sentence of the passage. In this sentence, the author of the passage draws a conclusion about the age of the Geminid stream. This conclusion is based on two pieces of information. The first is the length of time the Earth would take to cross the computer-model Geminid stream if the stream were 5,000 years old. The second is the actual elapsed time between the two peaks of meteor activity predicted by the computer model .In concluding from this information that the Geminid stream is actually only 3,000 years old, the author is assuming the accuracy of the computer model, as stated in E, the best answer. Choice A is incorrect because the passage says that the time the Earth takes to cross the stream would vary from year to year and that 19 hours was the average time, not the exact time, observed from 1970 to 1979. Choices B and C are incorrect because the passage does not suggest anything about the current state of the comet associated with the Geminid stream or about the expected longevity of the stream. Choices D is incorrect because the computer model is said to confirm the broadening predicted by the conventional theories; the fact that the model projected the positions of the particles in the stream over a 5,000-year period does not suggest that researchers expected the

CSAT Reading Comprehension

52 © Copyright 2011 Think & Learn Pvt. Ltd Copying or Reproduction without the prior permission of the author is prohibited

stream to be older (and therefore broader) than it turned out to be. 9. This question asks you to determine the main task that the passage is designed to accomplish. The best answer is B. The passage identifies a problem (shareholder’ and manager’ failure to enhance companies’ long-term productivity) in the first paragraph, most pointedly in the last sentence of that paragraph. In the second paragraph, the author recommends certain actions as a means of solving that problem. Choice A is not correct. The author of the passage identifies a problem in the first paragraph, but the author does not compare two different approaches to that problem. Rather, in the second paragraph, the author proposes a single, unified approach to solving the problem. Choice C is incorrect. The author does not defend an established method of institutional shareholding in the United States and recommends a different method in the second paragraph. Choice D is also incorrect. The author describes a situation in the first paragraph but does not provide data or draw any conclusions from data. Choice E is not the correct answer. The author does not compare alternative analyses of the current situation discussed in the passage. 10.This question asks you to decide what the passage implies, rather than states directly, about majority shareholders in a corporation. The best answer is C. According to the passage, those individual capitalists who were once majority shareholders in a corporation would not be able to make a quick profit by selling a large amount of stock because such a sale would depress the stock’s value. It can be inferred from the passage that this would be true of any majority shareholders. Choice A is not the correct answer. The passage suggests, that majority shareholders can actively influence a company’s decision-making, but it does not suggest that this influence is equal to the absolute authority suggested by the language of this answer choice. The passage also does not discuss the “operational management decisions” of corporations. Choice B is not correct. The passage does not specify what percent of a corporation’s stock any one shareholder is allowed to own. Choice D is not the correct answer. The author does not imply that majority shareholders are more interested in profits than in productivity. In fact, the author argues the opposite, stating that majority shareholders such as the old-style capitalists concentrated more on long-term productivity than on quick profits. Choice E is also incorrect. The author proposes that shareholders of more than 20 percent of a company’s stock should be required to give advance public notice before a stock sale, but the passage

does not suggest that majority shareholders are currently required to do so. 11. This question asks you to identify the stated purpose of the author’s suggestion that any institution holding 20 percent or more of a corporation’s stock be required to give the public one day’s notice of the intent to sell that stock. The best answer is C. The purpose of the requirement that institutions holding 20 percent or more of a company’s stock be required to give advance public notice of the sale of that stock is stated :to prevent insititutions from “trading shares at the propitious moment” and to encourage them to concentrate on increasing a company’s productivity. Choice A is incorrect. The suggested requirement that an institution give advance notice of its intent to sell a significant amount of stock tends to discourage institutional stockholders from selling stock they believe will decrease in value since, according to the passage, such an announcement would cause the stock’s value to plummet. Choice B is also incorrect. The author argues that insititutional stockholders should be “encouraged to take a more active role in the operations of the companies in which they invest.” The advance notice requirement discussed is proposed by the author as a means of fostering, not discouraging, institutional stockholders’ participation in the operation of the companies they invest in. Choice D is not the correct answer. The passage does not discuss ownership of company stock by that company’s employees. Choice E is also incorrect. The passage states only that institutions should be allowed to acquire “a dominant shareholding position in a corporation,” but it does not discuss whether investors should diversity their stock holdings by investing in different companies. 12.To answer this question, you must use information contained in the passage to infer something about the “old-style capitalist” referred to. The best answer is B. According to the lines of the passage, the individual capitalists of the past, referred to later in the passage as “old-style” capitalists , could not “sell out for a quick profit” because to do so would depress the value of their stocks. From this statement it can be inferred that someone who typifies the “old-style capitalist” would be unlikely to engage in short-term stock trading. Choice A is incorrect. A comparison between the old-style capitalists and their modern counterparts is made in the passage, but the passage does not express an opinion about whether or not the management techniques used by these capitalists are outdated. Choice C is not the correct answer. The passage does not discuss the investment policies of the corporations in which financial institutions invest. Choice D is also

CSAT Reading Comprehension

53 © Copyright 2011 Think & Learn Pvt. Ltd Copying or Reproduction without the prior permission of the author is prohibited

incorrect. According to the passage ,the old-style capitalists were individual investors, not large institutions. While the passage stats that large institutions are affected by anti-trust legislation, it says nothing about whether this legislation affects individual investors. In addition, the passage does not mention anything about how great a role individual investors now play in the stock market. Choice E is also incorrect. The passage states that the old-style capitalists focused on long-term productivity , and hence not on short-term profitability. 13. This question asks you to infer, from information stated in the passage, what the author assumes about the companies once controlled by individual capitalists. The best answer is B. The author asserts that individual capitalists “had to concentrate on improving the long-term productivity of their companies.” Then, a bit later, the author identifies improved long-term profitability as a consequence of improved long-term productivity. From this it can be inferred that the author assumes that if the businesses controlled by individual capitalists had improved long-term productivity, they would also have become more profitable. Choice A is not correct. The author does not make any direct comparison between the profitability of past and present corporations. Choice C is not the correct answer. The first sentence of the passage states that most large corporations were once dominated by individual capitalists who owned large portions of the companies’ stock, but the passage does not specify whether many or few people owned the remainder of each company’s stock. Choice D is incorrect. The passage states that the individual capitalists who once dominated large corporations “could not sell out for a quick profits,” but the passage does not indicate whether or not the other shareholders of these corporations were involved in short-term trading of their stock. Choice E is not correct. The passage does not suggest that intuitions owned no stock in most large corporations, only that individual capitalist owned enough stock to dominate these corporations. 14.This question asks you to infer, from information stated in the passage, a way in which the role of large institutions as stockholders differs from that of the “old-style capitalist.” The best answer is B. According to the passage, the old-style capitalists were able to play a dominant role in the corporations in which they held stock because they owned enough stock to do so. The passage also states that large institutions are legally barred from owning a majority of a company’s stock. From this it can be inferred that large institutions, because their ability to own stock is limited, do

not play as dominant a role in the corporations of which they are stockholders as did the old-style capitalists. Choice A is not correct. The passage does not indicate whether large institutions invest in many companies, few companies, or even just a single company. Choice C is not the correct answer. The passage does not mention brokers or any other parties who might influence the investment choices made by large institutions. Choice D is incorrect. The author notes that an institution’ sale of a large amount of stock would, in fact, decrease the stock’s value. Choice E is not the correct answer. The passage does not suggest any reason why large institutions are attracted to the stock of any particular corporations. 15.This question asks you to determine the main purpose served by the second paragraph of the passage in the context of the passage as a whole. The best answer is E. The second paragraph is devoted to the author’s recommendations of certain actions, namely, the adoption of new regulations concerning the holding and selling of stock by institutional investors. Choice A is not the correct answer. The author of the passage identifies a problem-short-term trading done by institutional shareholders-but does so in the first paragraph, not in the second. Choice B is incorrect. In the second paragraph, the author recommends new regulations regarding shareholding by institutions. Although the author mentions some consequences of the suggested regulations, the author clearly views these consequences as desirable. Choice C is not correct. The second paragraph contains suggestions about new ways to regulate shareholding by institutions. Although the author asserts that certain effects would result from the proposed regulations, these effects are not explained. Choice D is not correct. The second paragraph proposes measures to solve the problem presented in the first paragraph, but it simply describes, rather than evaluate, these proposed solutions.

TEST-RC 09

1. The best answer is D. In the first paragraph, the author outlines a business strategy, “developing integrated approaches for successful mass production and distribution.” The rest of the passage discusses the rivalry between Beta and VHS producers for control of the global VCR market, an illustration of the use of this strategy by VHS producers. The author briefly evaluates the two competing technologies and traces the impact of each on the market, but neither of these is the passage’s primary concern, so A and B are incorrect. Choices C and E are incorrect because no events are reinterpreted in the passage and no new approaches to business planning are proposed.

CSAT Reading Comprehension

54 © Copyright 2011 Think & Learn Pvt. Ltd Copying or Reproduction without the prior permission of the author is prohibited

2.The best answer is E. The passage sets up a contrast with “traditional” ways to benefit from marketing a product by stating that marketplace success based on leadership in “mass production and distribution” is characteristic of today’s companies, not those of the past. Choices B and D are incorrect because they describe ways in which companies have achieved success in the past. Choices A and C are incorrect: they present ways of earning profits that are not discussed in the passage. 3.The best answer is D. The passage states that the “perception among consumers that prerecorded tapes were more available in VHS format further expanded VHS’s share of the market.” None of the information given in the passage suggests that consumers thought the VHS-format was technically better (A) or less expensive than Beta (B). Nor does the passage indicate that consumers believed that VHS-format VCR’s were the first on the market (C) or that VHS-format VCR’s would eventually drive Beta VCR’s out of production entirely (E). 4.The best answer is D. VHS producers formed “strategic alliances with other producers and distributors” that helped manufacture and market their product, whereas Beta manufacturers “were reluctant to form such alliances and eventually lost ground” . Choice A is incorrect because it describes the strategy used by Beta producers. Choice B is incorrect because, although VHS producers held an early lead in sales, they did not give up their advantage. Choice C is also incorrect: the passage states that VHS manufacturers took advantage of “strategic alignments with producers of prerecorded tapes” , but no mention is made of a strict VHS monopoly on such tapes. Choice E incorrect because the passage does not suggest that VHS producers sacrificed technological superiority over Beta-format VCR’s. 5.The best answer is C. The alliances formed by VHS producers with videotape manufacturers created partnerships between companies whose products were mutually interdependent. Choice C, an alignment with the producer of a complementary product that is necessary for the original product to function correctly, offers the closest analogy. An alignment between manufacturers of competing products (A) is not analogous, nor is an alignment with a parts manufacturer (B). Choice D and E describe alignments with companies that distribute and market the product and are similar to some of the strategic alliances formed by VHS producers, but not to their alignment with videotape producers

6. The best answer is A. In the first paragraph, the author presents a general observation about contemporary business; the rest of the passage narrates a specific series of recent events concerning two companies in particular. The story of these companies serves as an example that illustrates the observation being made in the first paragraph of the passage. Choices B, C and E can be eliminated because the author does not advance an argument. Although the passage does advance an argument, the author does not anticipate that the conclusions being drawn will be disputed, so choice D is also incorrect. 7) (b) This is a data based question where the answer has to be got by running through the entire passage..Refer to the last but 5th line of the passage “Shaw, who has the courage of his historicism” and lo behold you will get the correct answer (b). 8) (b)This is really a tough question..It is an inference question and also an except question. First we need to find out which are the options Shaw would probably agree. Option (a) : look at the 4th line of the passage “the artist writes out of moral passion”. Shakespeare is an artist. So Shaw would agree to (a) Option (c) : Look at these line in the second paragraph “that the first great artist who comes along after a renewal gives to the new philosophy full and final form, that subsequent artists, though even more gifted, can do nothing but refine upon the master without matching him. Shaw, whose essential modesty is as disarming as his pose of vanity is disconcerting, assigns to himself the role, not of the master, but of the pioneer, the role of a Marlowe rather than of a Shakespeare” He indirectly points out that marlow is a pioneer and Shakespeare is a master. He also says that the master cannot be matched.So he will agree to option ( C ) Option (d ) : Look at these lines in the last paragraph “No one, says Shaw, will ever write a better tragedy than Lear” So Shaw will agree to option (d) also. So Option (b) is the correct answer. 9) (a) Refer to these line in the second paragraph “Shaw, whose essential modesty is as disarming as his pose of vanity is disconcerting” 10) (a) Another Tough Question. This is a data based question where the answer has to be got by running through the entire passage. Let us use POE to choose the correct answer. Option (B) Look at these lines “His teaching is that

CSAT Reading Comprehension

55 © Copyright 2011 Think & Learn Pvt. Ltd Copying or Reproduction without the prior permission of the author is prohibited

beauty is a by-product of other activity” He says that Beauty is a by product of other activities.But this option says that the drama is a product of beauty. So Shaw need not agree to this option. Option ( C ) “Morality is immutable” this statement is correct. But the second half of the option “it is not something to be discussed and worked out” is not correct. Look at these lines in the last paragraph “Shaw, who has the courage of his historicism, consistently withstands the view that moral problems do not change”.Shaw himself has discussed regarding these views. So Shaw need not agree to this option Option (d) Look at these lines in second paragraph “, it follows that even the best works of art go out of date in some important respects and that the generally held view that great works are in all respects eternal is not shared by Shaw” So Shaw need not agree to this option Option (A) is therefore correct. It is supported by the following lines in the first paragraph “; that the artist writes out of moral passion (in forms varying from political conviction to religious zeal)”

Test- RC 10

1. The best answer is C. According to the passage Randolph’s efforts to obtain recognition for the Brotherhood of Sleeping Car Porters, a goal achieved in 1935, “helped transform the attitude of Black workers toward unions.” The passage states that prior to 1935 that attitude was one of skepticism. Therefore, the passage indicates that the workers’ skepticism was diminished as a result of Randolph’s work. Choice A is incorrect because the passage does not indicate that the attitude among Black workers in other industries toward unions remained unchanged by 1935. Choice B is incorrect because the passage does not state that the actions of the Pullman Company’s union made Black workers more skeptical of unions by 1935. No information is given in the passage to support either D or E. 2.The best answer is D. In the passage, the author describes the skepticism of Black workers toward unions, “which had historically barred Black workers from membership,” as “understandable,” thus conveying an appreciation that the attitude of Black workers had a legitimate basis. Choices A and B can be eliminated because they concern Randolph and the Brotherhood rather than the attitude of the workers mentioned in passage. Choice C is incorrect because the author expresses no ambivalence about the

significance of unions to Black workers. Choice E is incorrect because the word “understandable” is used in the passage to describe the attitude of the Black workers rather than the historical attitude of unions toward Black workers. 3.The best answer is B. The passage states that the existence of a Pullman-owned union “weakened support among Black workers for an independent entity” such as Randolp’s Brotherhood of Sleeping Car Porters. Thus, the passage suggests that some porters did support the Pullman Company union. The passage directly contradicts A, which implies virtually no support of the Pullman company union. Choice C is incorrect because the passage does not suggest anything about Pullman employees other than porters. Choice D is incorrect: in fact, the passage suggests that the porters’ attitude was less positive after Randolph’s union achieved recognition in 1935.Choice E is incorrect because the passage nowhere suggests that the general skepticism of Black workers had no effect on their response to the Pullman Company’s union. 4.The best answer is C. The passage indicates that it was an advantage to Randolph in building the Brotherhood’s membership that Black workers shared “the same grievances from city to city”, suggesting that it would have been more difficult to build the membership of the Brotherhood if Black workers in different parts of the country had had different grievances. There is no information in the passage to support A, B, D, and E. 5.The best answer is A. The passage indicates that indicate that in the early 1930’s it became illegal for a company to maintain its own union with company funds. Thus, the passage suggests that prior to the 1930’s a company was permitted to fund its own union. Choices B, C, D, and E describe practices that are not implied in the passage. 6.The best answer is B. The passage states that Randolph’s reason for brining the Brotherhood into the American Federation of Labor: “as a member union, the Brotherhood would be in a better position to exert pressure on member unions that practiced race restrictions.” No information is given in the passage to support choices A, C, D and E. 7.The best answer is B. The passage describes a marketing strategy practiced by some professional service firms, outlines the arguments in favor of its use, and the describes the drawbacks associated with the strategy. Choice A is incorrect because the popularity of the practice is not discussed in the passage. Choice C is incorrect because the passage does not include a demonstration of how to

CSAT Reading Comprehension

56 © Copyright 2011 Think & Learn Pvt. Ltd Copying or Reproduction without the prior permission of the author is prohibited

institute unconditional guarantees. Choice D is incorrect because ethical issues are not addressed by the author. In the first paragraph, the author does give reasons for why firms pursue the strategy, but that is not the purpose of the passage as a whole, so E is also incorrect. 8.The best answer is A. The passage explicitly states that offering unconditional guarantees works best with first time clients, but no mention is made of the relative success of this practice with clients of long standing. The circumstances referred to in B, C, D, and E are mentioned in the passage. 9.The best answer is B. In the passage the author states that firms offering unconditional guarantees are “seeking a competitive advantage.” Choices A, C, and E can be eliminated because no mention is made in the passage of liability limits, justification for fee increase, or improvement of service as objectives in offering unconditional guarantees. Choice D is incorrect because the passage does not associate the offering of unconditional guarantees with the attainment of an outstanding reputation. 10.The best answer is B. The passage states that a guarantee “may mislead clients by suggesting that lawsuits or medical procedures will have guaranteed outcomes”. This implies that legal and medical outcomes cannot be predicted with absolute certainty. Although the statements in A, C, D, and E could be true, none of these statements is implied by the description in the passage. 11.The best answer is D. The passage states that an unconditional guarantee “may, paradoxically, cause clients to doubt the service firm’s ability to deliver the promised level of service.” In D, an architect’s unconditional guarantee leads prospective clients to wonder whether previous clients have been satisfied with that architect’s work, a situation that exemplifies the paradox described in the passage. Choice A describes a problem that would occur after a service has been rendered. Similarly, in C, a guaranteed service has actually been judged unsatisfactory. Choice B exemplifies the situation described in the passage: guarantees may suggest that a firm is “begging for business.” Choice E is incorrect because it presents a situation that is the reverse of the potential problem mentioned in the passage. 12.The best answer is E. The passage states that “professional service firms with outstanding reputations and performance to match have little to gain from offering unconditional guarantees.” If a firm has an outstanding

reputation based on a high level of performance, it is likely that its clients are satisfied with the quality of service that is delivered. Choice A is not implied: the passage indicates that the author believes that firms with outstanding reputations have little to gain from offering unconditional guarantees, but the passage implies nothing about whether such firms either currently offer guarantees or have offered them in the past. The passage does not provide information concerning the profitability (B) or the fees charged by highly reputed firms (D). In addition, it does not suggest that such firms practice in fields in which outcomes are predictable (C).